Insgesamt ist Thomsons Modell ziemlich beeindruckend. Mit nur wenigen Annahmen konnte sie eine ganze Menge Dinge plausibel erklären – insbesondere die Struktur des Periodensystems konnte zumindest prinzipiell plausibel gemacht werden. Allerdings muss man auch sagen, dass das Modell seine Schwächen hat – die Rechnungen wurden eben nur für den zweidimensionalen Fall gemacht, bei dem die Elektronen auf Ringen umlaufen, so dass ein direkter Abgleich mit dem Experiment nicht möglich war. Trotzdem diente es doch für eine ganze Weile als taugliches Atommodell (auch wenn einige Veränderungen notwendig wurde als man erkannte, dass ein Atom deutlich weniger Elektronen enthält als Thomson ursprünglich dachte – dafür wurde dann die positive adung als massebehaftet angenommen).

Was können wir heute daraus lernen?

Das Thomson-Modell war insofern ein gutes Atommodell, als es von Annahmen ausging, die mit allem vereinbar waren, was man damals sicher wusste. Es war auch insofern ein gutes Modell, weil es genau definierte Annahmen enthielt, die es erlaubten, zumindest modellhafte Rechnungen durchzuführen. (Ohne diese Rechnungen hätte Thomson ja auch nicht nachweisen können, dass die Theorie prinzipiell gut zum Periodensystem passt.)

Man kann dem Modell natürlich ankreiden, dass es eben nicht wirklich für den Fall von Elektronen auf dreidimensionalen Schalen berechnet wurde – aber letztlich ist das ja nicht die Schuld von Thomson. Wenn die Natur nun mal aus Teilchen aufgebaut ist, die sehr komplexes Verhalten zeigen, dann kann es eben sein, dass man eine Theorie zwar hinschreiben, aber nicht lösen kann. (Das Problem hat ja zumindest teilweise die Theorie der Quarks und Gluonen.) Im Rahmen ihrer Möglichkeiten hat Thomson das gezeigt, was eben ging. (Hätte Thomson schon Computer gehabt, hätte sie das System vermutlich numerisch simulieren können.)

Weiterhin wurde das Modell später widerlegt. (Rutherford hat übrigens auch ein “bisschen mehr” gemacht als nur zu sagen “Oh, wenn einige Teilchen beim Bestrahlen einer Goldfolie sehr stark abgelenkt werden, dann muss da wohl ein sehr kleines Teilchen im Atom sitzen” – auch hier ist die Arbeit voll mit detaillierten Rechnungen.) Auch das ist eine Stärke des Modells – es macht genügend spezifische Vorhersagen, dass man es durch Experimente widerlegen kann. Und das liegt eben daran, dass das Modell sauber formuliert wurde und innerhalb des Modells genau klar war, wie sich die einzelnen Elektronen verhalten.

Falls ihr also eure eigene Theorie aufstellen wollt, wie die Welt funktioniert, lernt von Thomsons Modell, obwohl oder gerade weil es sich am Ende als falsch herausstellte:

1. Sind die einzelnen Bestandteile eures Modells und deren Verhalten wohldefiniert? Habt ihr Gleichungen dafür, was die Modellbestandteile tun und wie sie sich verhalten? Wisst ihr, wie das Verhalten der Bestandteile mit Messgrößen zusammenhängt? Vage Ideen wie “Die Welt besteht aus Raumzeitwirbeln” oder “Die Raumzeit ist in Wahrheit ein dynamisches Punktgitter” sind keine Theorie, sondern genau das: vage Ideen.

2. Habt ihr die Konsequenzen des Modells durchgerechnet? Ich bekomme ständig mails der Art “Ich habe hier ne tolle Theorie, wie die Welt funktioniert, aber leider fehlt mir die Mathematik, um sie durchzurechnen und zu beweisen, dass sie stimmt.” Tja – solange ihr nicht berechnet habt, was die Konsequenzen eurer Theorie sind, taugt sie recht wenig.

3. Ist eure Theorie konsistent mit dem, was wir wissen (und wir wissen ne Menge, siehe oben)? Thomsons Modell wäre sicher nie akzeptiert worden, wenn es nicht so gut zum Periodensystem und den bekannten Eigenschaften der Atome gepasst hätte (auch wenn die Übereinstimmung nur qualitativ war).

4. Gibt es offensichtliche Schwachstellen oder Lücken (sowie die Bindung gleichartiger Atome bei Thomson oder auch die Ideen zur Radioaktivität, die doch ein bisschen problematisch erscheinen)? Darüber solltet ihr nachdenken und diese Schwachstellen auch deutlich erwähnen.

5. Selbst wenn eure Theorie durchgerechnet ist und einigermaßen vernünftig aussehende Vorhersagen macht (so wie die von Thomson) – sie kann immer noch falsch sein. In den Worten von Huxley:

the great tragedy of Science — the slaying of a beautiful hypothesis by an ugly fact

1 / 2 / 3 / 4 / 5

Kommentare (146)

  1. #1 Grobian
    28. Juli 2015

    Kommentar gelöscht.
    Deine Homophobie kannst du anderswo verbreiten.

  2. #2 Herr Senf
    28. Juli 2015

    “In Zukunft dient deshalb die Kommentarspalte dieses Posts als Tummelplatz für alle, die gern Privattheorien verkünden wollen. (Erwartet aber nicht, dass ich die alle kommentiere…)”
    Und von wer(*) wird erwartet, irgendwelchen Murks zu lesen 🙂

  3. #3 Bjoern
    28. Juli 2015

    Danke für die Besprechung von Thomsons Arbeit! 🙂 Ich kannte bisher nur die Veröffentlichungen von Schrödinger und Rutherford zu ihren Atommodellen.

    Hier scheint aber ein Tippfehler zu sein:

    Allerdings ist die Stabilität bei 6 Atomen labil …

    Das sollte wohl “Elektronen” heißen?

  4. #4 Phil
    28. Juli 2015

    Text zum Thema feminine Formen gelöscht

  5. #5 dgbrt
    28. Juli 2015

    Text zum Thema feminine Formen gelöscht

    Aber zum Thema:
    Ich habe in meiner Jugend die Obergrenze Lichtgeschwindigkeit angezweifelt. Aber mir war auch bewusst, dass Albert Einstein eigentlich Recht haben sollte. Also habe ich dann die Herleitungen und Formeln der Speziellen Relativitätstheorie kennen gelernt und auch verstanden.
    Das können aber vermutlich schon die Wenigsten, da geht dann einfach nur die Phantasie durch. Es gibt tolle Ideen, aber Wissenschaft ist das natürlich nicht.

    Physikerinnen und Physiker sollten es sich also zur Aufgabe machen, ihre Ergebnisse so zu präsentieren, dass die Mehrheit der Menschen, ohne Kenntnisse der höheren Mathematik, das trotzdem verstehen und akzeptieren können.

    Einstein hat das immer mit Gedankenexperimenten gemacht. Da braucht man kein Mathe, aber wenn wirklich ernsthaft darüber nachdenkt, kann es jeder verstehen. Das Prinzip wird heute viel zu selten in der Öffentlichkeitsarbeit verwendet.

  6. #6 ulfi
    28. Juli 2015

    Ich finde es sehr beeindruckend, was Frau Thomson mit dem wenigen, was wir ueber Atome wussten gemacht hat um ein Modell vorzuschlagen, welches zumindest ein paar Beobachtungen erklaeren kann.

    Und man sieht auch, was das fuer sie fuer eine Arbeit gewesen sein muss, all die Beobachtungen erklaeren zu koennen. Heutzutage wird das Vorstellen einer neuen Theorie damit aber viel schwieriger, weil es eine vielzahl von Beobachtungen gibt. Aber was erwartet man auch? Wenn man das Universum erklaeren will, hat man viele Dinge zu zeigen.

  7. #7 MartinB
    29. Juli 2015

    @Bjoern
    Danke für den Hinweis

    @alle
    Wer sich über meine Grammatik aufregen will, kann das anderswo tun – entsprechende Blogtexte mit Kommentarspalten existieren ja zu Hauf.

    Alles, was hier zu dem Thema gesagt wird, werde ich löschen (und habe es auch schon gelöscht).

    Und für alle, die das ganz doll schlimm finden:
    https://scienceblogs.com/insolence/files/2015/01/CheeseWhine.jpg

  8. #8 MartinB
    29. Juli 2015

    @dgbrt
    ” Da braucht man kein Mathe, aber wenn wirklich ernsthaft darüber nachdenkt, kann es jeder verstehen. ”
    Letztlich ist das aber ein teil des Problems, das ich hier in diesem text anspreche: Wenn man Physik so vermittelt, dann entsteht eben der Eindruck, man müsse sich nur ein hübsches Gedankenexperiment ausdenken und das sei dann schon Physik.

  9. #9 AldiGuru
    29. Juli 2015

    Diesen Artikel muss ich wirklich mehr als loben, einer der besten die ich in der Vergangenheit auf SB gelesen habe!

    Einerseits spannend die Entwicklung des Modells nachzulesen und andererseits wirklich wichtig um all die Physikphilosophen, Esoteriker und Weltbildveränder zu entlarven.

    Als i-Tüpfelchen habe ich auch noch eine Idee über den Nutzen des generischen Femininums erhalten, von dem ich bisher eher skeptisch war (wenn auch nicht ablehnend). Ich werde mich daran versuchen, das auch öfters zu verwenden.

    Bleibt nur noch eine Frage: Hat Einstein in der Arbeit zur ART auch die Grenzen der Theorie erwähnt? Was wären die Grenzen dann in dem Falle?

  10. #10 MartinB
    29. Juli 2015

    @AldiGuru
    Danke für’s Lob.
    Das mit Einsteins Arbeit weiß ich ehrlich gesagt nicht – da habe ich nie in die Originalpapers im Detail reingeschaut. Soweit ich weiß war ein Problem, dass Einstein ja (außer in Grenzfällen wie Lichtablenkung oder Periheldrehung) keine Lösung der Gleichungen hatte – die Schwarzschild-Metrik hat ja Schwarzschild gefunden. Insofern war es vermutlich schwer, Grenzen zu sehen. (Auch heute hat die Theorie ja wenige Grenzen – auf der einen Seite die Vereinbarkeit mit der QM, auf der anderen Seite Singularitäten, aber beides war 1916 kein Thema.) Hauptproblem war ja anscheinend eher, dass das Universum nicht stabil wäre.

  11. #11 CM
    29. Juli 2015

    Teil des Problems ist sicher auch, dass es an Universitäten und Forschungseinrichtugen nicht wenige leitende Persönlichkeiten gibt, deren wissenschaftliches Denken schon lange einen Realitätsabgleich braucht. Wer kennt sie nicht, die Professoren, die sich auf allen möglichen Feldern für kompetent halten und mit der Zeit bereits die Kompetenz im eigenen Feld verloren haben (wenn nicht in der eigenen Gruppe, dann woanders)?

    Nicht wenige Experimente werde somit auf Anordnung ausgeführt, was nicht schlecht sein muß, doch manches Mal fehlt es schlicht an logischer Stringenz …

    Danke für den Beitrag!

  12. #12 Krypto
    29. Juli 2015

    @AldiGuru, MartinB:
    Einstein hat auch Lösungen gefunden, aber eben nicht als Erste.
    Wie Martin schon erwähnt hat, war es Einsteins Hauptproblem, dass sich ohne Weiteres kein statisches Universum berechnen ließ. Und das war eine vermutete Unverträglichkeit mit den damals verfügbaren Beobachtungsdaten(=Experimenten), bis Edwin Hubble die Galaxienflucht entdeckt hatte.

  13. #13 Krypto
    29. Juli 2015

    Hat jemand eigentlich schon ArthurB bescheid gegeben, dass sie sich hier austoben darf? 😀

  14. #14 schlappohr
    29. Juli 2015

    Leider begehen viele Autoren/Dozenten/Lehrer den Fehler, die Behandlung eines Themas auf reine Mathematik zurückzuführen. Ich erinnere mich an Unterrichtstunden und Vorlesungen, deren Ergebnis mit Herleitungen vollgeschriebene Tafeln waren, aber die längst keinen Bezug mehr zum Thema hatten. Der Dozent stand kurz vor Ende der Vorlesung durchgeschwitzt aber glücklich an der Tafel, weil die erwartete Gleichung herausgekommen war. Der eigentliche Kern der Sache, nämlich die Interpretation der Gleichungen, wurde schlichtweg vergessen. Ich habe noch einige grausige Exemplare von Büchern aus meinem Studium im Regal stehen, die fast keinen Text enthalten, sondern nur Gleichungen. Auch wenn man sich die Mühe macht, diese Gleichungen detailliert nach zu vollziehen, ist das daraus erwachsende Verständnis praktisch Null. Wenn man die natürliche Sprache vollständig durch Formeln ersetzt, geht der wichtigste Teil der Information verloren. In einigen Übungen im Studium gab es sogar Punkteabzug, wenn man es wagte, neben den Formeln noch einige erklärende Worte hin zu schreiben, z.B. wie der Beweis strukturiert ist und welche Idee dahinter steht (wenn der Korrektor die Herleitung nicht nachvollziehen konnte, gab es natürlich auch Abzug).

    Die Mathematik ist das zentrale und absolut unverzichtbare Werkzeug in der Wissenschaft, aber sie ist nicht deren Zweck (außer natürlich in der Mathematik als Wissenschaft selbst, aber das ist eine ganz andere Sache). Insofern finde ich es zumindest für die Wissensvermittlung sehr kontraproduktiv, sich auf reine Mathematik zu beschränken. Offenbar sehen das einige Wissenschaftler genauso, daher auch die erwähnten Gedankenexperimente von Einstein, und Feynman sagt in seinen Lectures (zumindest in dem Teil, den ich bisher gelesen habe), dass er mit Mathematik eher zurückhaltend umgehen wird. Es gibt kaum eine Seite ohne Gleichungen, aber anstatt diese seitenweise herzuleiten, verwendet er viel Text, um die Bedeutung der Ergebnisse für das Thema klar zu machen. Ich muss allerdings gestehen, dass es Fälle gibt, in denen die Herleitung selbst für das Verständnis essenziell ist.

    Ich weiß nicht, wie es anderen geht, aber bei mir persönlich hat die starke Fokussierung auf die Mathematik das Verständnis für die Sache oft blockiert. Es ist so, als müsste ich den Umgang mit einem Musikinstrument bis zur Virtuosität erlernen, ohne jemals Einblick in die Partitur zu bekommen. Auch als ich später selbst Übungen betreut habe, ist mir oft aufgefallen, dass viele Studenten die Mathematik zwar beherrschten, aber offensichtlich keine Ahnung hatten, was sie da eigentlich ausrechneten. Das wurde immer dann besonders deutlich, wenn ich sie fragte, was es denn für das eigentliche Problem bedeute, wenn eine Lösung plötzlich komplex wird oder in einem Anhängigkeitsgraph Zykel auftreten.

    Um nun den Bogen zum Thema zu schlagen: Ich glaube, dass genau daraus bei vielen eine Aversion gegen die Mathematik entsteht. Mathematik gilt als ein unglaublich kompliziertes Gebäude aus Formeln, deren Zweck verborgen bleibt. Also wird versucht, intuitiv Wissenschaft zu betreiben, was dann oft zu drolligen Ergebnissen und Privattheorien führt.

  15. #15 Benjamin Packisch
    Ein schöner Artikel, aber..
    29. Juli 2015

    ..leider gibt es prinzipiell keine Theorie, die ALLES, was wir physikalisch wissen, “korrekt” wiedergibt. Jede Theorie beruht auf einer spezifischen Faktenauswahl und -wertung, denn es gibt (noch) keine Weltformel.
    Die Auswahl bleibt aber oft aus praktischen Gründen implizit, und genau das macht den Dialog zwischen In- und Out-Physikern oder Fachferneren so schwierig.
    Beide Seiten beziehen sich auf unterschiedliche, womögliche disjunkten Faktensammlungen, und auf dieser Ebene könnte man sehr viel voneinander lernen.

    Bei der Betonung der Fakten wird es allerdings heikel.
    Denn schon zwischen den Physik-internen Spezialdisziplinen gibt es abweichende Faktengewichtungen. Die fallen üblicherweise nicht so sehr auf, weil der Spezialisten-übergreifende Dialog sich innerhalb der Physik sich häufig eher im PR-Bereich abspielt, mit wenig Bereitschaft zu Grundstürzendem.

    Das kann dann zu realen und vermeintlichen Widersprüchen im Schulphysikgebäude führen, die von Innen und Außen unterschiedlich gut sichtbar sind.

    Wie sehr man diese Widersprüche nun betont, ist wohl eher eine Charakterfrage und weniger eine der Fachausbildung.
    Ein Beispiel: Die (mathematische) Quantifizierung des Sagnac-Effekts, wie sie im Laserkreisel verwendet wird.
    Die technische Numerik ist sehr simpel und klassisch auf einer absoluten Lichtgeschwindigkeit beruhend. Die physikalische state-of-the-art-Numerik ist
    dagegen voll ART-konform und ziemlich mühselig, aber es gibt sie, auch wenn durchaus nicht alle Physiker sie wirklich durchsteigen. Der Sagnac-Effekt selber wird in einem durchschnittlichen Physik-Studium auch nur am Rande erwähnt, wenn überhaupt.
    Daraus folgt eine gruselige Konfliktlage, bei denen zwei Seiten sich jeweils redlich Idiotie vorwerfen dürfen, ohne irgendetwas für die Wissenschaft zu erreichen.

    Global betrachtet in der Physik gibt es nicht wenige solche Spannungslagen, je weiter man den Fachhorizont spannt, desto mehr, und desto einsamer. Nun ist es dann
    wieder eine Charakterfrage, ob man diese Widersprüche der eigenen, womöglich defizitären Detailkenntnis zuschreibt, oder ob man das Phänomen als fundamental bezeichnet,
    und sich selbst als genial.
    Den Teufel nun mit dem Beelzebub austreiben zu wollen, d.h. als Fachfremder mit Genieanspruch allen eine eigene, voraussetzungsreiche Theorie als “Universallösung”
    aufzudrängen ist natürlich verstiegen und nutzlos.
    In der Schulung zur Demut sehe ich ehrlich gesagt den größten Nutzen eines Physikstudiums.
    Otto Wiener halte ich dabei für ein sehr positives Beispiel zu dieser Spannung zwischen Genie und Demut.

    In jedem Falle begrüße ich jeden Versuch, wie auch den hier von MartinB, einen wachen und ehrlichen Dialog zwischen den Fronten anzustoßen. Denn allein eine
    Garantie auf wissenschaftliches Recht gibt es nicht und für niemanden. In manchen Fällen weiß der Narr besseren Rat als der amtliche Weise und er erkennt
    des Kaisers neue Kleider deutlicher – manchmal!

  16. #16 hampel
    29. Juli 2015

    Sehr schöner Text, danke.

    So etwas bräuchten wir meiner Meinung nach viel mehr und früher in den Schulen, dass könnte vielleicht auch helfen die Probleme , die @schlappohr anspricht , zu umgehen.

    Mir ist persönlich damals die Lust an Physik in der Schule schnell vergangen , da sie in meinen Augen nur noch aus Mathematik bestand. Heute bereue ich total, nicht selbst mehr getan zu haben und muss nun mühsam einfachste Grundlagen neu lernen, “nur” um mein Hobby Astronomie besser zu verstehen. Es war mir aber ehrlich gesagt damals unmöglich, sämtlichen Formeln und Berechnungen zu folgen UND nebenbei noch über deren mögliche Anwendungen nachzudenken. (Eine durschnittliche Physikunterrichtstunde bestand aus 2×2 vollgeschriebenen Tafeln, die der Lehrer noch selber geputzt hat -damit die Tafel schneller wieder zur Verfügung steht. .)

    Vielleicht stimmt aber auch der Spruch :
    ‘When you get into university, you learn that Biology
    is really Chemistry, Chemistry is really Physics and Physics is Math (and Math is really hard)” 😉

    Völlig OT aber was mich heute noch wundert:
    Sämtliche Erkenntnisse zum Thema “Atom” z.B. Schalen- und Orbitalmodell und auch Radioaktivät wurden damals im Chemieunterricht abgehandelt, ist das normal bzw heute noch so?

    Zum Text selber , Punkt 3 scheint mir im Zusammenhang mit den Erfahrungen im Quarks-Beitrag bemerkenswert

    konsistent mit dem, was wir wissen

    Wenn man annimmt, dass schon seit Newton alles falsch gemacht wird in der Physik , bürdet man sich wirklich eine große Aufgabe auf.

  17. #17 MartinB
    29. Juli 2015

    @schlappohr
    Ich stimme dir vollinhaltlich zu – das fängt schon in der Schule an, wo man im wesentlichen Formeln hin- und herschiebt und dann Zahlen einsetzt.
    Ich denke eben nur, dass der rein populärwissenschaftliche Weg genau die Gefahr birgt, die ich hier im Text kurz beschrieben habe. Deswegen versuche ich im Blog ja auch des öfteren, einen Mittelweg zu finden und die Mathematik selbst anschaulich zu machen (Maxwell-Serie, QFT-Serie usw.).

  18. #18 schlappohr
    29. Juli 2015

    @MartinB

    ” Deswegen versuche ich im Blog ja auch des öfteren, einen Mittelweg zu finden […]”

    Ja, und das gelingt Dir ganz hervorragend, so hätte ich mir das im Studium gewünscht.

    Der “formelfreie” Weg führt in der Wissenschaft natürlich nicht zum Ziel (*). Deswegen hinterlassen die typischen populärwissenschaftlichen Bücher (auch wenn sie wirklich gut sind, z.B. von Randall oder Greene) immer einen faden Nachgeschmack bei mir. Aber in der “harten” Fachliteratur ertrinkt halt manchmal das eigentliche Thema in der Mathematik.

    (*) Wobei manche das anders sehen. Ich weiß nicht, ob Du das Machwerk hier kennst, leg Dir auf jeden Fall eine Aspirin zurecht: https://www.ekkehard-friebe.de/dogmen.html.

  19. #19 Benjamin Packisch
    29. Juli 2015

    @Schlappohr:
    Das Problem ist vor allem, dass es keine eindeutige Abbildung zwischen Mathematik und Physik gibt, AUßER derjenigen, die durch Konvention stabilisiert ist.
    Die Konventionen werden ihrerseits stabilisiert durch die Mühsal, die sich jedem ins Gedächtnis brennen, der sie während eines Studium lernt. Da will man ntendenziell gerne einen “return-of-invest” sehen und lässt sich nicht gerne reinpfuschen von Leuten, die diese Schmerzen nicht hinter sich gebracht haben.

    Mathematisch-formale Dressur ist aber nicht gleichbedeutend mit mathematisch-physikalischem Überblick. Das kann so sein, und es ist am leichtesten in Prüfungen messbar. Aber nur weil es gut messbar ist, beweist es nicht zwingend physikalische Kompetenz. Formaler Irrsinn ist sehr gut möglich, genauso wie intuitives Physik-Genie in Bildern möglich ist.

    Eine einfache Haltung zur Unterscheidung der beiden gibt es leider nicht.

  20. #20 schlappohr
    29. Juli 2015

    Ich denke, man kann die Physik schon eindeutig durch Mathematik beschreiben, natürlich außer den Aspekten die man noch nicht kennt oder versteht, und jenen, die man bewusst weg lässt, weil sie keine relevante Rolle spielen (Stichwort effektive Theorien).

    “Mathematisch-formale Dressur ist aber nicht gleichbedeutend mit mathematisch-physikalischem Überblick”.

    Exakt, da besteht kein Zusammenhang, auch wenn die Dressur im Allgemeinen ihre Vorteile hat. Ein rein intuitives Begreifen der Physik halte ich auch für Genies für unmöglich, spätestens, wenn es um Quantenmechanik geht. Ich weiß nicht ob das stimmt, aber ich meine gelesen zu haben, dass Planck die Grundlagen der Quantenmechanik entdeckte, in dem er mit der Rayleigh-Jeans-Gleichung auf einem Blatt Papier “herumspielte” und erst dadurch auf den Trichter kam, dass man eine Quantisierung benötigt. Manchmal ist die Mathematik der einzige Weg zur Lösung, aber der Link zur Realität sollte eben immer sichtbar bleiben.

  21. #21 MartinB
    29. Juli 2015

    @Benjamin
    ” und lässt sich nicht gerne reinpfuschen von Leuten, die diese Schmerzen nicht hinter sich gebracht haben. ”
    Naja, Fakt ist aber nunmal, dass man moderne Experimente ohne einen haufen Mathematik nicht korrekt beschreiben und vorhersagen kann.

    @sclappohr
    Wobei für Planck die Quantisierung zunächst nur eine Annahme war, die das rechnen erleichterte – Planck war ja kein Anhänger der Atomhypothese und glaubte an konituierliche materie; er hatte nur die Idee, dass es der Hohlraumstrahlung egal sein sollte, wie sie erzeugt wird, und das rechnen wurde so einfacher. (Quelle für diese Aussage habe ich irgendwo rumfliegen, aber gerade nicht griffbereit.)

  22. #22 Karl Mistelberger
    29. Juli 2015

    > #9 AldiGuru, 29. Juli 2015
    > … Bleibt nur noch eine Frage: Hat Einstein in der Arbeit zur ART auch die Grenzen der Theorie erwähnt? Was wären die Grenzen dann in dem Falle?

    Clifford Will hat seinen schon etwas älteren Übersichtsartikel auf den neuesten Stand gebracht:

    The status of experimental tests of general relativity and of theoretical frameworks for analyzing them is reviewed and updated. Einstein’s equivalence principle (EEP) is well supported by experiments such as the Eötvös experiment, tests of local Lorentz invariance and clock experiments. Ongoing tests of EEP and of the inverse square law are searching for new interactions arising from unification or quantum gravity. Tests of general relativity at the post-Newtonian level have reached high precision, including the light deflection, the Shapiro time delay, the perihelion advance of Mercury, the Nordtvedt effect in lunar motion, and frame-dragging. Gravitational wave damping has been detected in an amount that agrees with general relativity to better than half a percent using the Hulse–Taylor binary pulsar, and a growing family of other binary pulsar systems is yielding new tests, especially of strong-field effects. Current and future tests of relativity will center on strong gravity and gravitational waves.

    The Confrontation between General Relativity and Experiment

  23. #23 Strudel
    29. Juli 2015

    Sehr interessant, ich hatte bisher echt keine Ahnung was im Thomsonschen Atommodell so alles drin steckt. Ein gutes Beispiel, welche Rolle Modelle in der Wissenschaft spielen.

    Aber eine Frage habe ich: Thomson war doch ein Mann, oder irre ich mich? Ich wollte eigentlich einem Freund, der Physik unterrichtet, einen Link auf diesen Artikel schicken. Der Text wirkt aber durch die feminine Form etwas skurril, was er doch eigentlich gar nicht verdient hat.

  24. #24 Krypto
    30. Juli 2015

    @ Strudel: Auch auf die Gefahr hin, dass Dein und mein Kommentar gelöscht werden, möchte ich einen Vorschlag an die Autorin machen:
    Wie wäre es, wenn Du öfter mal Vornamen in Deinem Text verwendest? 30xThomson ohne ihren Vornamen…

  25. #25 MartinB
    30. Juli 2015

    @Strudel
    Wie schon gesagt, siehe meine Texte zum Thema Femininum im Blog

    @Krypto
    Könnte ich machen. Aber ist es nicht letztlich vollkommen egal, ob Thomson ein Mann war oder eine Frau?

  26. #26 schlappohr
    30. Juli 2015

    “Aber ist es nicht letztlich vollkommen egal, ob Thomson ein Mann war oder eine Frau?”

    Nein. In diesem Fall muss ich Dir widersprechen, aus den folgenden Gründen:

    1. Wenn Du einen Namen erwähnst, kommt damit eine reale Person ins Spiel, die in den meisten Fällen entweder weiblich oder männlich ist. Auch wenn das für die Tatsache, dass die Person einen entscheidenden Beitrag zur Wissenschaft geleistet hat, keine Rolle spielt, der Leser möchte sich üblicherweise dennoch ein Bild von der betreffenden Person machen, insbesondere, wenn es sich um einer Person mit einer gewissen historischen Bedeutung handelt.

    2. Du musst im Allgemeinen davon ausgehen, dass die Person selbst, in diesem Fall Thomson, Wert darauf legt (bzw. legen würde), in der korrekten Form genannt zu werden. Die meisten Menschen mögen es nicht, wenn ihre charakteristischen persönlichen Merkmale falsch dargestellt werden. In meinem Fall wurde einmal im Anhang einer Veröffentlichung versehentlich ein Foto einer anderen Person abgedruckt, was ich als sehr unangenehm empfunden habe, obwohl es sachlich betrachtet keine Rolle gespielt hat.

    2. Wenn Thomson wirklich eine Frau gewesen wäre, so wäre das durchaus bemerkenswert, da es in den Naturwissenschaften ja bedauerlicherweise nur wenige Frauen gibt, und diese in jedem Fall eine besondere Würdigung verdienen, besonders im historischen Kontext des frühen 20. Jahrhunderts. Auch wenn das für den wissenschaftlichen Beitrag wiederum keine Rolle spielt.

  27. #27 MartinB
    30. Juli 2015

    @schlappohr
    “der Leser möchte sich üblicherweise dennoch ein Bild von der betreffenden Person machen,”
    Und aus irgendeinem Grund ist das Geschlecht die wichtigste Information über einen Menschen. Genau das ist ja der Grund, warum ich generische Formen mag – sie hinterfragen genau diese Annahme, dass Menschen primär durch ihr Geschlecht definiert sind.
    “Die meisten Menschen mögen es nicht, wenn ihre charakteristischen persönlichen Merkmale falsch dargestellt werden”
    Ich stelle niemanden falsch dar – ich verwende die grammatikalisch weibliche Form für alle, also exklusiv und generisch – damit sollte dann auch klar sein, dass über das Geschlecht keine Aussage getroffen wird.

    “Wenn Thomson wirklich eine Frau gewesen wäre, so wäre das durchaus bemerkenswert, da es in den Naturwissenschaften ja bedauerlicherweise nur wenige Frauen gibt”
    Das ist in der Tat der Punkt, der mich seinerzeit ein wenig ins Stolpern gebracht hat (und das auch immer noch tut) – wenn man nur noch weibliche Formen verwendet, und das Geschlecht nie erwähnt, schmälert man dann die Leistung der Frauen, die es damals gegen alle Widrigkeiten geschafft haben, in der Wissenschaft eine Leistung zu vollbringen?
    Nach einigem Nachdenken bin ich zu dem Schluss gekommen, die Analogie z.B. zur Hautfarbe zu sehen – würde ich die Hautfarbe Thomson’s erwähnen, wenn Thomson nicht weiß gewesen wäre (was ja in der damaligen Zeit ebenfalls sehr bemerkenswert gewesen wäre)? Wenn ja, dann könnte ich in vergleichbarer Situation auch das Geschlecht erwähnen (ich würde also bei einem Artikel über Emmy Noether durchaus erwähnen, dass sie eine Frau war, weil es für die Bewertung ihrer leistung in dem Fall wichtig sein mag). Warum für diese Hervorhebung aber unbedingt eine grammatikalische Form gebraucht wird, erschließt sich mir nach wie vor nicht.

    Falls du das weiter diskutieren willst, bitte nicht hier, sondern da:
    https://scienceblogs.de/hier-wohnen-drachen/2015/03/26/das-generische-femininum/

  28. #28 Benjamin Packisch
    30. Juli 2015

    @schlappohr:
    Trotz Anweisung auf Thread-Umzug von MartinB eine kurze Anmerkung zu Deinem m.E. guten Einwand:
    Die offensive “sie”-Verwendung ist schlicht Geschlechts-Relativierung, und sie folgt damit dem gleichen Geist, wie einst die Feier der Relatitivitätstheorien: Nicht absolute Qualia sollen die Welt “sein”, sondern nur relative Bezüge.

    Das ist erkenntnistheoretisch auch löblich, weil im Prinzip zutreffend. ABER: Es gibt einen Unterschied zwischen dem Bezug zwischen einem selbst (dem Beobachter) und einem Objekt, und zwischen der Beobachtung eines Bezugs zwiscnen zwei Objekten.
    Wenn man also nur noch Bezüge und keine monadischen Qualia wissenschaftlich gelten lassen will, dann ist das möglich, aber logisch korrekt nur wenn man den Bezug zwischen Subjekt und Objekt nicht mit dem zwischen Absolut und Relativ verwechselt. Eine Geschwindigkeitsdifferenz zweier Objekte, die ich beobachte, ist etwas anderes als die v-Differenz eines der Objekte zu meinem v.

    Otto Wiener hat das 1921 sehr schön in seiner Strömungsäther-Theorie ausgeführt (die NB nicht RT-kritisch ist).

    Grüße
    BP

  29. #29 MartinB
    30. Juli 2015

    @Benjamin
    Irgendwie verwendest du seltsame Begriffe – was sollen denn “absolute Qualia” sein? Qualia sind doch gerade durch ihre Subjetikvität definiert.

  30. #30 Benjamin Packisch
    30. Juli 2015

    @MartinB
    Okay, vielleicht kein ganz glücklicher gewählter Qualia-Begriff oben von mir, ich meinte ihn im Sinne bewußtheitlicher Qualitäten mit “Universalien”-Anspruch. Klassisch: im Sinne platonischer Ideen. (siehe dazu ausführlichUniversalienproblem, aber das führt auf eine andere Baustelle, oder?)

  31. #31 MartinB
    30. Juli 2015

    @Benjamin
    Sorry, aber mit der Erklärung wird es für mich auch nicht verständlicher – da stecken zu viele nicht deutlich formulierte Annahmen drin. Warum sollte es solche Platonischen Ideen geben? Wie soll es “nur Bezüge” geben – zwischen was sollen das Bezüge sein?

    Und ich bitte nochmal darum, Debatten zu meinem Sprachgebrauch hier beim entsprechenden Text zu führen und nicht hier, wo sie nicht hingehören.

  32. #32 Karl Mistelberger
    30. Juli 2015

    Frau Bäker: Thomson

    Wikipedia: Sir Joseph John “J. J.” Thomson

    Frau Bäker: Ende des 19. Jahrhunderts, noch bevor die Atomtheorie wirklich vollkommen anerkannt war, wusste man schon, dass Atome (wenn es sie denn gibt) Bestandteile haben müssen, als 1897 das Elektron entdeckt wurde.

    Wikipedia: Thomson, in 1897, was the first to suggest that one of the fundamental units was more than 1,000 times smaller than an atom, suggesting the subatomic particle now known as the electron. Thomson discovered this through his explorations on the properties of cathode rays. Thomson made his suggestion on 30 April 1897 following his discovery that cathode rays (at the time known as Lenard rays) could travel much further through air than expected for an atom-sized particle. He estimated the mass of cathode rays by measuring the heat generated when the rays hit a thermal junction and comparing this with the magnetic deflection of the rays. His experiments suggested not only that cathode rays were over 1,000 times lighter than the hydrogen atom, but also that their mass was the same in whichever type of atom they came from. He concluded that the rays were composed of very light, negatively charged particles which were a universal building block of atoms.

    Frau Bäker: Thomson nahm an – so lesen wir meist – dass die Elektronen in einem gleichmäßig positiv geladenen Medium steckten, das kontinuierlich war. Man nennt es deswegen auch gern Rosinenkuchen-Modell, weil die kleinen deutlich unterscheidbaren Elektronen wie Rosinen in einem Teig aus positiver Ladung sitzen.

    Wikipedia: In 1904 Thomson suggested a model of the atom, hypothesizing that it was a sphere of positive matter within which electrostatic forces determined the positioning of the corpuscles. To explain the overall neutral charge of the atom, he proposed that the corpuscles were distributed in a uniform sea of positive charge. In this “plum pudding” model the electrons were seen as embedded in the positive charge like plums in a plum pudding (although in Thomson’s model they were not stationary, but orbiting rapidly).

    Frau Bäker: Insgesamt ist Thomsons Modell ziemlich beeindruckend.

    The Nobel Prize in Physics 1906: Thomson returned to America in 1904 to deliver six lectures on electricity and matter at Yale University [Silliman memorial lectures]. They contained some important suggestions as to the structure of the atom.

    Beeindruckend ist, was J.J. Thomson in der experimentellen Physik geleistet hat. Dafür wurde ihm 1906 der Nobelpreis verliehen. Ebenso beeindruckend ist der Erfolg seiner Mitarbeiter: Sie und sein Sohn erhielten insgesamt acht Nobelpreise.

    Zurecht eher vergessen ist sein Atommodell, auch wenn Frau Bäker davon ziemlich beeindruckt ist.

  33. #33 Benjamin Packisch
    31. Juli 2015

    @MartinB:
    Gegenfrage: Was spricht Deiner Meinung gegen platonischen Ideen?
    Ein Beispiel: das Dreieck. Es gibt kein ideales auf der Welt, trotzdem weiß (fast) jede/r, welche realen Symbole eine Dreieck meinen. Wenn ein Kind einen Baum malt, dann malt es keinen realen Baum, sondern es manifestiert sein Ideal davon.
    Woher kennen wir aber diese Idealisierungen? Eine mögliche und verbreitete Position ist: Man lernt das aus Dressur/Erziehung. Dann bleibt aber die Frage: Wann fing das an mit der Dressur? Und woher kam es?

    Auch Dein Artikel und seine Forderung nach mathematischer Formulierung einer (physikalischen) Theorie setzt eine Mitteilbarkeit nicht-physischer Zusammenhänge voraus. Ohne qualitatives Richtig-Falsch-Empfinden jenseits der Physik wäre es unmöglich bzw. sinnlos, mathematische Theorien zu verbreiten.

    Die Frage ist, ob dieses “Richtig-Falsch-Empfinden” reine, akademische Konditionierung ist. Wenn ich Dich richtig verstehe, ist das Deine Meinung, oder?

    P.S.: Mit Bezügen ist gemeint: das Relativitätsprinzip der Logik, dass also etwas Reales (z.B. Messung) nur durch seinen Bezug zwischen zwei Einheiten gegeben ist (z.B. Messobjekt und Messgerät). Das Konzept “nacheinander” ist ein Beispiel. dafür: es wird erst sinnvoll durch einen willkürlich gesetzten BEZUGSpunkt in der Zeit.

  34. #34 MartinB
    31. Juli 2015

    @Benjamin
    Dass wir aus der Umwelt Abstraktionen ableiten, bedeutet ja nicht, dass diesen Abstraktionen eine “Realität” außerhalb unseres Bewusstseins zukommt.
    Warum sollte das so sein? Und wann wird ein Abstraktum zur platonischen Idee? Und warum sollten dann unterschiedliche Menschen unterschiedliche Ideen vieler Abstrakta haben? Oder sollen nur “einfache “Konzepte wie “Dreieck” platonisch sein, aber komplizierte Konzepte wie “Demokratie” oder “unendlichdimensionaler Hilbertraum” sind es nicht? Wo soll dann die Grenze verlaufen?
    “Auch Dein Artikel und seine Forderung nach mathematischer Formulierung einer (physikalischen) Theorie setzt eine Mitteilbarkeit nicht-physischer Zusammenhänge voraus. Ohne qualitatives Richtig-Falsch-Empfinden jenseits der Physik wäre es unmöglich bzw. sinnlos, mathematische Theorien zu verbreiten.”
    Habe ich nicht verstanden.

    “Mit Bezügen ist gemeint: das Relativitätsprinzip der Logik, dass also etwas Reales (z.B. Messung) nur durch seinen Bezug zwischen zwei Einheiten gegeben ist (z.B. Messobjekt und Messgerät). ”
    Dass etwas Reales nur durch einen Bezug “erkannt” werden kann – o.k. Dass aber deswegen alles, was real ist, nur durch Bezüge definiert ist, ist nicht dasselbe.

  35. #35 Benjamin Packisch
    31. Juli 2015

    @MartinB
    1. … Oder sollen nur “einfache “Konzepte wie “Dreieck” platonisch sein, aber komplizierte Konzepte wie “Demokratie” oder “unendlichdimensionaler Hilbertraum” sind es nicht? Wo soll dann die Grenze verlaufen?
    Die Grenzziehung ist allerdings nicht ganz einfach, aber ich denke sie ist nicht der wesentliche Aspekt. Wesentlich ist, dass Konzepte zwar nicht physisch real sind, aber doch real wirksam. Das führt auf die Frage, ob denn unser abstraktes Denken etwas Reales ist, oder “nur” eine Illusion, oder auch “nur” z.B. ein Willensimpuls.
    Ich denke, es gibt sowohl konditionierte, von außen eingeübte Konzepte, als auch a priori bzw. intuitiv zugängliche Konzepte, und beide können wirksam sein. Eine Grenzziehung dazwischen ändert nichts an der Wirklichkeit beider Pole. Auf der einen Seite überwiegt der Manipulationswille, auf der anderen der Erkenntniswille.

    2. …Dein Artikel … setzt … Mitteilbarkeit nicht-physischer Zusammenhänge voraus -> Habe ich nicht verstanden.
    Ich meine schlicht, dass es ohne Annahme eines nicht-konditionierten, d.h. a priori oder intuitiv feststehenden Grundkonsenses nichts bringt, Physik mathematisch mitzuteilen. Dass sowas überhaupt erlernbar ist, beweist m.E., dass es Formen gibt, die intuitiv wirken.

    3. Dass etwas Reales nur durch einen Bezug “erkannt” werden kann – o.k. Dass aber deswegen alles, was real ist, nur durch Bezüge definiert ist, ist nicht dasselbe.
    Verstehe ich jetzt nicht ganz. Kannst Du vielleicht ein Beispiel geben für etwas Reales, das ohne Bezug definiert ist?

  36. #36 MartinB
    31. Juli 2015

    @Benjamin
    “ob denn unser abstraktes Denken etwas Reales ist, oder “nur” eine Illusion, oder auch “nur” z.B. ein Willensimpuls.”
    Das ist eine falsche Dichotomie – ein Computerprogramm ist auch “real”, ohne dass es materiell wäre.
    “Ich meine schlicht, dass es ohne Annahme eines nicht-konditionierten, d.h. a priori oder intuitiv feststehenden Grundkonsenses nichts bringt, Physik mathematisch mitzuteilen.”
    Siehe dazu auch hier:
    https://scienceblogs.de/hier-wohnen-drachen/2011/08/15/ist-die-natur-mathematisch/

    @Benjamin
    “Kannst Du vielleicht ein Beispiel geben für etwas Reales, das ohne Bezug definiert ist?”
    Das ist insofern (wie ja auch angedeutet) schwierig, weil zum Messen eines Objekts immer ein Bezug da sein muss – letztlich ist es aber die alte Frage, ob ein Baum ein Geräusch macht, wenn niemand im Wald ist, um es zu hören.
    Irgendwo im Weltall schwebt bestimmt ganz einsam im intergalaktischen raum ein Proton, das seit Äonen mit keinem Teilchen wechselgewirkt hat (zumindest ist das denkbar). Beginnt das erst dann zu existieren, wenn ich es messe?

  37. #37 Benjamin Packisch
    31. Juli 2015

    @MartinB:
    “ob denn unser abstraktes Denken etwas Reales ist, oder “nur” eine Illusion, oder auch “nur” z.B. ein Willensimpuls.”
    Das ist eine falsche Dichotomie – ein Computerprogramm ist auch “real”, ohne dass es materiell wäre.

    Ich habe das auch nicht behauptet, im Gegenteil. Es ist beides real, nur in unterschiedlichen Realitätsausprägungen – die eben nicht dichotom, sondern, wenn in Bezug zueinander, komplementär sind.

    Irgendwo im Weltall schwebt bestimmt ganz einsam im intergalaktischen raum ein Proton, das seit Äonen mit keinem Teilchen wechselgewirkt hat (zumindest ist das denkbar). Beginnt das erst dann zu existieren, wenn ich es messe?
    Wenn nein, dann müsste man seine Existenz absolut (d.h. bezugssystemsunabhängig) und objektiv (d.h. von jedem Bezugssystem aus potentiell messbar) annehmen. Es wäre dann gewissermaßen nicht wegtransformierbar…

    Die eigentlich Frage war ja, ob man ohne Bezug überhaupt definieren kann. Und ich meine nein. Mindestens muss man einen Bezug negieren. Zum Beispiel den der Relevanz einer Relativgeschwindigkeit zweier Protonen, die ich beobachte von meinem rotierenden Stuhl aus z.B. 😉
    Grüße und eine schönes WE
    BP

  38. #38 MartinB
    31. Juli 2015

    “ob man ohne Bezug überhaupt definieren kann.”
    Naja, das wusste letztlich schon Kant – “das ‘ich denke’ muss alle meine Vorstellungen begleiten können”, IIRC.
    Ein nicht zusammengesetztes Objekt ohne Bezug kann nicht interagieren, also kann es auch nicht wahrgenommen werden. Das ist letztlich trivial. Das heißt aber nicht, dass ein Objekt bloß die Summe aller seiner Relationen ist – weil es einfacher sein mag, ein Proton als Proton zu beschreiben statt als Summe aller denkbaren reaktionen, die einem Proton möglich sind.

  39. #39 erik||e
    1. August 2015

    @MartinB
    “Ein nicht zusammengesetztes Objekt ohne Bezug kann nicht interagieren, also kann es auch nicht wahrgenommen werden. Das ist . . . ..”

    Isaac Newton ist auch an seinem Denken “gescheitert”, seine “Mechanik” mit der Alchemie zu Einem Ganzen zu verbinden. Physikerinnen sind immer noch bestrebt zu verstehen, wie das Wort (Information, Software – keine Materie …) mit den Formen der Materie zu Einem Ganzen zusammen zu fügen ist . . . ..

    Ist es doch unser Denken, welches uns nicht erlaubt zu verstehen, was uns umgibt . . . .. Welches Medium steht dem Verstehen gegenüber? actio = reactio . . . .. alle drei sollten wohl |=0| ergeben . . . ..

  40. #40 MartinB
    1. August 2015

    @erik||e oder wie auch immer
    Zur Internethöflichkeit gehört aber generel, sich einen eindeutigen Nicknamen zuzulegen (das macht es mir dann auch leichter, deine Texte zu überspringen – so muss ich immer gucken, ob es vielleicht ein anderer Erik ist, der etwas sinnvolles schreibt). Kannst du dich also bitte entscheiden, unter welchem Namen du hier posten möchtest und dann dabei bleiben?
    Danke.

  41. #41 erik||e oder wie auch immer . . . ..
    1. August 2015

    @MartinB
    . . . .. das soll mein Name sein!
    Danke.

  42. #42 Benjamin Packisch
    3. August 2015

    @MartinB:
    … Ein nicht zusammengesetztes Objekt ohne Bezug kann nicht interagieren, also kann es auch nicht wahrgenommen werden. Das ist letztlich trivial.
    Das heißt aber nicht, dass ein Objekt bloß die Summe aller seiner Relationen ist – weil es einfacher sein mag, ein Proton als Proton zu beschreiben statt als Summe aller denkbaren reaktionen, die einem Proton möglich sind.

    Jetzt wird es spannend. Wenn ein Objekt als NICHT bloß die Summe aller möglichen Relationen und Interaktionen ist, dann MÜSSEN wir etwas analog dem platonischen Ideenreich annehmen.

    Genauer: wir müssen also über-empirische Seinsbehauptungen oder Idealisierungen an den Anfang unserer physikalischen Modellbildung stellen.

    Und schon kommen wir zu wirklich wesentlichen Fragen:
    Zum Beispiel: Wie diskutiert man in der Physik die überempirischen Bestandteile der Theorien?
    Oder:
    Wie vollständig sind die gängigen Abstraktionen über ihre empirischen Interaktionsmöglichkeiten definiert und wo wirken dort möglicherweise überempirische Postulate (z.B. im E-Feld)?

    Ich denke hinter diesen Fragen wohnen nicht eben kleine Drachen, die sicher nicht nur einen Artikel wären…

  43. #43 Krypto
    5. August 2015

    @Benjamin:
    Du trägst ein wenig zu dick auf.
    Es mag wohl eine Frage sein, die Dich sehr interessiert, aber ob das eine tragfähige Mehrheit von Physikinteressierten so wesentlich findet…?

  44. #44 Benjamin Packisch
    5. August 2015

    @Krypto:
    Es geht in der Wissenschaft nicht Mehrheit, sondern um Wahrheit, mein Guter. Und wenn das hier nur Wissenschafts-PR ohne Interesse an echten Fragestellungen, dann bitte ich um expliziten Hinweis, z.B im Artikel: “Vorsicht, Mehrheitsfang!” oder “Hier geht es um Unterhaltung, nicht um Physik”.
    Du verstehst ;-)?

  45. #45 Benjamin Packisch
    5. August 2015

    +um
    +ist
    [bitte oben eingefügt denken]

  46. #46 Krypto
    5. August 2015

    @Benjamin:
    Ich glaube, Du verstehst mich nicht richtig:
    Du wirfst mal eben in den Ring:
    “Und schon kommen wir zu den wirklich wesentlichen Fragen:”
    Damit möchtest Du(Wer bist Du überhaupt?) allen hier im Blog Aktiven aufdiktieren, was _wirklich wichtig_ ist?
    …naja…

  47. #47 Benjamin Packisch
    6. August 2015

    @Krypto:
    Gerne darfst Du auch zur Sache etwas sagen. Denn die dürften jeden Wissenschaftler und Freund der Wissenschaft zuallererst interessieren.
    Ich will niemandem etwas diktieren, ich stelle nur etwas zur Disposition, was ich in MartinBs Artikel für sachlich relevant halte – und bislang für unterbetont – neben all den bereits erwähnten relevanten Aspekten. Meine Person ist dazu wirklich unwesentlich – Deine übrigens auch – wenn überhaupt irgendeine, dann die von MartinB.

    Wenn jemand also sachliche Einwände hat oder Anregungen – bitte jederzeit gern.

  48. #48 MartinB
    18. August 2015

    @Benjamin
    ” Wenn ein Objekt als NICHT bloß die Summe aller möglichen Relationen und Interaktionen ist, dann MÜSSEN wir etwas analog dem platonischen Ideenreich annehmen.”
    Warum? Letztlich ist ein “Objekt” immer eine Abstraktion – weil es in der wirklichen Welt zum Beispiel nicht eindeutig ist, welches Atom gerade zum Tisch gehört und welches nicht (an der Oberfläche werden sich ja z.B. mal Moleküle kurz anlagern und dann wieder lösen usw., an einer rauen Stelle mag sich ein Faden verfangen, von dem ein Teil dann tief in einem Riss stecken bleibt, der Kaffeklecks mag eine Verfärbung im Holz zurücklassen usw. – und am Ende gibt es eh nur ein Quantefeld für alle Elektronen usw., das weiß auch nix von “Tischen” und ihren Grenzen).
    “Objekte” sind also letztlich immer gedacht.
    Und natürlich kann man Objekte nur durch Interaktionen wahrnehmen – das heißt aber nicht, dass die einfachste Beschreibung der Welt nur aus Interaktionen besteht (und unsere Alltagserfahrung zeigt, dass wir Dinge wie “Tische” auch ohne Interaktionen als Objekte annehmen).
    Warum soll daraus schon folgen, dass es eine platonische Ideenwelt gibt?

  49. #49 miesepeter3
    20. August 2015

    @ MartinB

    “Dann wird ein bisschen herumphilosophiert und -argumentiert; meist vollkommen ohne jede Mathematik….”

    Wenn ich mich an meine eher mittelstufenmäßige Schulbildung richtig erinnere, gab es schon etwa 400 vChr einen Herrn Demokrit, der da so rumphilosophierte, dass es da wohl so etwas wie ein unteilbares Irgendwas geben müßte, aus dem die Welt besteht. Dabei war besonders schwer vorstellbar, dass dieses Ding fast nur aus leeren Zwischenräumen besteht. Er hat es nach meiner Erinnerung verhältnismäßig verständlich ausgedrückt und hat dafür gar keine Mathe benutzt. War seine Theorie deswegen Schei…benhonig?

  50. #50 MartinB
    20. August 2015

    @miesepeter
    Jedenfalls war es keine physikalische Theorie im heutige Sinn, sondern letztlich nur eine Spekulation, die soweit ich weiß auch keienrlei empirische Grundlage hatte.
    Ja, Demokrit hatte recht – aber nicht jede, die recht hat, hat aus den richtigen Gründen recht. Die Frage ist, welche Methode einigermaßen verlässlich zu korrekten Ergebnissen führt, und da schneidet “hinsetzen und nachdenken” nicht so gut ab, was die Beschreibung der physikalischen Welt angeht. Die Griechen hatten eben keine Möglichkeit, zwischen Atomlehre oder der Idee “Alles ist Wasser” und ähnlichen Ideen der Vor-Sokratiker zu entscheiden.

  51. #51 miesepeter3
    20. August 2015

    @ MartinB

    Nein, natürlich war das noch keine richtige Theorie, aber ein Erklärungsversuch der Welt ohne Mathe. Erst nach seiner Wiederentdeckung wurde in diese Richtung geforscht und irgenwann fand irgendwer heraus, dass Demokrit nicht weit genug gedacht hat, Sein Unteilbares war noch weiter teilbar
    (siehe Atombombe). Aber sein Verdienst war es, dass diese Vorstellung sich überhaupt entwickelte. Die Mathematik als Beweismittel für das eine oder andere scheint mir immer nur modellhaft zu funktionieren. Für fast jede physikalische Theorie gibt es einen mathematischen Beweis, und für jede Theorie eine Gegentheorie incl. mathematischem Gegenbeweis.
    Meine geringen mathematischen Kenntnisse stoppen hier meine Ausführungen, aber so weit ich weiß, gibt es sogar eine Matheformel, bei der immer zwei gegensätzliche Ergebnisse herauskommen sollen. Da wird meine Verständnis über Beweiskraft auf eine ziemlich harte Probe gestellt.

  52. #52 MartinB
    20. August 2015

    @miesepeter
    “Für fast jede physikalische Theorie gibt es einen mathematischen Beweis”
    Das ist quatsch – nur, die, für die es keinen Beweis gibt, bekommt man normalerweisenicht zu sehen (außer manchmal in den Kommentarspalten dieses Blogs…)

    “und für jede Theorie eine Gegentheorie incl. mathematischem Gegenbeweis.”
    Nein. Es gibt weder zur z.B. Relativitätstheorie noch zur Quantenmechanik o.ä, eine Gegentheorie mit Gegenbeweis. Physik ist nicht beliebig.

    “aber so weit ich weiß, gibt es sogar eine Matheformel, bei der immer zwei gegensätzliche Ergebnisse herauskommen sollen. ”
    Nein. Es gibt die Gödelschen Sätze, aber das ist ne ganz andere Baustelle…

  53. #53 a.n
    20. August 2015

    Was ist denn bitte ein mathematischer Beweis einer physikalischen Theorie?

    “Die Mathematik als Beweismittel […]”
    Mathematik kann überhaupt nichts beweisen, das nicht mathematisch ist. “Mathematik als Beweismittel” funktioniert tatsächlich nur modellhaft, weil nur mathematische Modelle mathematisch sind.

  54. #54 MartinB
    20. August 2015

    @a.n.
    Ich denke mal (wohlwollend interpretiert), miesepeter meinte, dass man eine theorie mathematisch so beschreibt (hintrickst), dass die Ergebnisse mit den Messungen übereinstimmen.

  55. #55 Thorsten Heitzmann
    20. August 2015

    @ # 52 :
    https://scienceblogs.de/deutsches-museum/2014/04/15/zeitmessung-mit-schiefer-ebene/#comment-139

    Mit korrekter Mathematik und beschriebenen experimentellen Beweis.

    Mir ist unverständlich warum Jemand einen Fakt ignoriert.

  56. #56 Krypto
    21. August 2015

    @Thorsten:
    Einen Kommentar 139 sehe ich nicht, sondern nur 4 ziemlich dumme Ergüsse von Taber.

  57. #57 Benjamin Packisch
    21. August 2015

    @MartinB:
    ” Wenn ein Objekt als NICHT … die Summe aller … Interaktionen ist, dann MÜSSEN wir etwas analog dem platonischen Ideenreich annehmen.”
    Warum? Letztlich ist ein “Objekt” immer eine Abstraktion ..
    Und wodurch meinst Du erhalten unsere Abstraktionen denktechnische Stabilität? Dein Beispiel mit uneindeutig definierten Atomen verdeutlicht m.E. sehr gut, dass die Empirie logisch unvollständig ist: Obwohl wir nicht mal ein konkretes Atom spezifizieren können, ist die physikalische Betrachtungsweise regelmäßig so, “als ob” wir es könnten; bzw. so, “als ob” Atome nicht Idealisierungen wären, sondern materielle Konkreta.

    Wir können also mindestens die physische Realität von Idealisierungen vortäuschen. Die Frage ist: warum bedient sich die Physik dieser Täuschung so wirksam?

    Ich halte den Schluss für durchaus konservativ, daraus die Realität einer objektiven Außenwelt zu folgern, an die wir uns – warum auch immer – durch Idealisierungen annähern können.
    Bezogen auf den Artikel lautet dann eine weterführende Frage: Wie sollten die überempirschen Ideal-Bestandteile einer Theorie in der Physik am besten diskutiert werden?

  58. #58 MartinB
    21. August 2015

    @Benjamin
    “Ich halte den Schluss für durchaus konservativ, daraus die Realität einer objektiven Außenwelt zu folgern, an die wir uns – warum auch immer – durch Idealisierungen annähern können.”
    Ja, das ist o.k. – warum diese Welt “platonisch” sein soll (also es eine zusätzliche abstrakte Ideenwelt geben soll), erschließt sich mir aber nicht. Kann so sein, aber dafür sehe ich keinen Anhaltspunkt.
    “Wie sollten die überempirschen Ideal-Bestandteile einer Theorie in der Physik am besten diskutiert werden?”
    So wie wir es jetzt auch tun – als Modellbestandteile einer Theorie. Wie sonst?

  59. #59 Benjamin Packisch
    21. August 2015

    @MartinB
    Ich glaube wir nähern uns an. Konsens ist also, dass die physikalische Modellbildung Kategorien erfordern, die unserer Erfahrung nicht direkt zugänglich sind – egal ob man die jetzt platonisch, transzendent oder überempirisch nennt. Ich meine zwar schon konkrete Anhaltspunkte für geometrische Grundprimitive in der Natur zu sehen, etwa Parabel und goldener Schnitt, aber gut, dass mag man auch einfach mathematisch nennen und nicht “Ideen”.

    Aber jwerden wir doch mal physikalisch etwas konkreter, und zwar in der Frage nach der Hierarchie in den Idealisierungen. Da sehe ich einen deutlichen Wertungs-“Bias” zugunsten der Mainstream-Lehrmeinung, der mich belustigt, wo er transparent ist, der aber meine nüchterne Physikerseele entsetzt, wo er vernachlässigt oder geleugnet wird.
    Es gibt m.E. viele Beispiele, und je mehr ich in dieser Richtung suche, desto mehr finde ich. Konkretes Beispiel:

    Die Berechnung der Phasenverschiebung im Laser- oder Faserkreisel, einem Standardinstrument der Luftfahrt, das auf dem Sagnac-Interferomenter beruht. Mit der Annahme eines ruhende Äthers (einer Idealisierung) ein Vierzeiler, mit dem Werkzeugkoffer von SRT und ART (einer weiteren Idealisierung) ein Vierseiter – bei identischem Ergebnis.

    Doch die Diskussion dieser Koinzidienz sucht man vergebens, sowohl inner- als auch außerakademisch, obwohl sie m.E. in höchstem Maße instruktiv wäre.
    Ersteres sei eine Näherung ist die Schul-Überzeugung, und zweiteres richtiger. Ersteres ist aber keine Näherung, weil es die Messwerte nicht schlechter oder besser als zweiteres beschreibt- sondern genauso.
    Daraus muss ich schließen, dass es eine Hierarchie konventionell akzeptierter Ideen gibt, die nicht konsequent durch die Empirie oder Occams Rasiermesser gesteuert ist.

    Die platonische Ideenwelt ist es offenbar nicht Deiner Meinung nach. Was aber dann?
    Mir fallen da nur wenig schmeichelhafte Gründe ein, und insofern würden mich GUTE Gründe außerordentlich interessieren, sowohl für diesen Einzelfall, als auch für dessen Allgemeinheit, die z.B. manchem Doktoranden bekannt sein dürfte.

  60. #60 MartinB
    21. August 2015

    @Benjamin
    ” egal ob man die jetzt platonisch, transzendent oder überempirisch nennt.”
    Du bist in meinen Augen mit Begriffen mehr als schlampig – eine platonische Ideenwelt ist eine Welt *außerhalb* und *zusätzlich* (so wie z.B. bei Penrose beschrieben – in der echten Platonischen Lehre gibt es ja ideale Konzepte von z.B. “Pferd”, von denen real existierende Pferde gewissermaßen abgeleitete Instanzen sind (um es mal programmiersprachenmäßig auszudrücken)). Das ist nicht dasselbe wie etwas transzendentes (das eine zusätzliche nicht direkt beobachtbare Eigenschaft unserer Welt sein kann) oder etwas, dass nicht direkt e,mpirisch nachweisbar ist (im positivistischen Sinne, wie etwa eine Wellenfunktion in der Kopenhagener Deutung der QM). Das alles in einen Topf zu werfen, macht die Diskussion nicht gerade einfacher.

    “Konkretes Beispiel:…”
    Das Beispiel kenne ich nicht, insofern kann ich inhaltlich nicht viel dazu sagen. Aber wie man in der Philosophie sagt “Aus falschem kann man beliebiges folgern” – es ist also nicht überraschend, dass man auch mal eine falsche Theorie findet, die eine richtige Vorhersage macht, und zwar sogar einfacher. Das gilt schon für die Aristotelische Mechanik – die erklärt viel “einfacher”, warum ein Ball irgendwann zur Ruhe kommt, wen ich ihn rolle, ohne mühselige Konzepte wie “Reibung” o.ä.
    Dass das ein Argument für irgendwas ist, sehe ich nicht.

    “obwohl sie m.E. in höchstem Maße instruktiv wäre.”
    Dann diskutier das doch – die meisten Leute werden das halt nicht instruktiv finden, aber da du doch Physikerin bist, hindert dich doch niemand, das mal sauber zu durchdenken und aufzuschreiben, wenn was interessantes dabei rauskommt. “Warum schreibst du nicht über das, was ich wichtig finde” ist ein Vorwurf, den ich auch als Bloggerin öfters bekomme – kann ich nur mit der Schulter zucken. Forschung ist frei, und für eine rein mathematische Betrachtung der beiden Herleitungen, die dich interessieren, brauchst du nicht mal Geld von der DFG.

    “Daraus muss ich schließen, dass es eine Hierarchie konventionell akzeptierter Ideen gibt, die nicht konsequent durch die Empirie oder Occams Rasiermesser gesteuert ist. ”
    Nein, daraus musst du schließen, dass man zur Akzeptanz einer Theorie nicht bloß ein Phänomen nimmt, sondern alle. Und die SRT erklärt nun mal einen ziemlichen Haufen Phänomene, von der Chemie über die Kernphysik bis hin zu Astronomie udn Teilchenphysik. (Siehe meinen entsprechenden Blogtext.) Und unter diesen quasi tausenden von Erklärungen gibt es jetzt eine, die man mit einer anderen “Theorie” leichter herleiten kann (aber mit demselben Ergebnis), während diese andere “Theorie” die anderen Ergebnisse nicht herleiten kann. Und dass ist jetzt relevant weil…? Mich überzeugt das nicht, um meine Zeit mit ner Äthertheorie zu verbringen – aber wenn es dich überzeugt, dann tu’s doch und sag’ uns dann was dabei rauskommt.

    Ein besseres Beispiel, bei dem ich dir viel eher folgen würde, ist die Dominanz der Kopenhagener Deutung der QM, die ja nicht besser ist als andere Interpretationen. Aber da sind sich eben auch alle einig, dass die Deutungen nur eine Frage der Interpretation sind, weil sie alle dieselben Vorhersagen machen. Und es gibt eben keine Antwort auf die Frage, welche der Deutungen die “Realität” besser beschreibt.

    “sowohl für diesen Einzelfall, als auch für dessen Allgemeinheit, die z.B. manchem Doktoranden bekannt sein dürfte.”
    Der Satz ist grammatikalisch irgendwie vermurkst – worauf bezieht sich das “dessen” (auf Einzelfall – macht wenig sinn?), worauf bezieht sich das “die” (auf die Allgemeinheit? Aber wovon?).

  61. #61 Benjamin Packisch
    22. August 2015

    @MartinB:
    Ohweh, ich wollte Dir nichts für den Blog vorschreiben! Die Existenz einer Kommentarseite und Deine rege Reaktion darauf lädt eben zur kontroversen Diskussion ein, und ich lerne sehr viel dabei – und Du vielleicht auch zumindest etwas über (manche) Deine Leser ;-)? Ich denke, wenn wir beide höflich bleiben, ist da durchaus Spielraum.

    Zur Sache: Dir mißfällt offenbar meine Analogisierung von platonischer Ideenwelt und überempirischen Bestandteilen einer Theorie – nun gut, ich will letztlich nur das ausdrücken, was Du programmiersprachenmäßig beschreibst: Die Spannung zwischen Klasse und Instanz. Eine Klasse (oder ein Ideal/Theoriekonzept) müssen jeweils nicht instanziiert (realisiert/messbar) sein, um denktechnisch wirksam zu sein, das Prinzip ist immer das gleiche.
    Konkret in einem japanischen Sprichwort: Wenn man als einziges Werkzeug einen Hammer hat, sieht die ganze Welt aus wie ein Nagel.

    Du schreibst, dass man zur bestmöglichen Wahl einer theoretischen Idealisierung “alle Phänomene” heranziehen müsse. Ich denke, das ist selbst ein Ideal, kein schlechtes zwar, aber auch keines, dass wir schon irgendwo erreicht hätten. Du erwähnst die QM, die ich auch für ein gutes Beispiel halte für die Deutungsfreiheit (im Prinzip wohl das gleiche ist wie bei der Faserkreisel-Numerik oder in vielen anderen technisch genutzten Physikbereichen). Die Frage ist also: Welche Theorie sollten wir wählen, wenn formal und prognostisch kein Unterschied besteht? Die einfachste? Die anschaulichste? Die mathematischste? Die umfassendste? Die üblichste?

    Das “dessen” bezog sich auf “Einzelfall”, weil ich kühn einen Einzelfall als zu einer ungenannten Allgemeinheit gehörig behauptet habe. Ich wollte damit so dezent wie möglich den “Autoritätsbeweis” zur Theoriewahl andeuten, der Doktoranden mitunter begegnet. Wie schätzt Du die Wirksamkeit von “Autoritätsargumenten in der Wahl der besten Theorie ein?

  62. #62 MartinB
    23. August 2015

    @Benjamin
    “Ich denke, wenn wir beide höflich bleiben, ist da durchaus Spielraum. ”
    Ehrlich gesagt, keine Ahnung, was Höflichkeit damit zu tun hat und woher du die Notwendigkeit für diese Bemerkung siehst. Gelernt habe ich bisher nicht viel aus unserer Diskussion, um ehrlich zu sein.
    “Dir mißfällt offenbar meine Analogisierung von platonischer Ideenwelt und überempirischen Bestandteilen einer Theorie ”
    Wenn du das nur als Analogie siehst, o.k., aber das wurde bisher nicht deutlich. maW: Mir misfällt, dass du dich extrem unpräzise ausdrückst.

    “. Ich denke, das ist selbst ein Ideal, kein schlechtes zwar, aber auch keines, dass wir schon irgendwo erreicht hätten.”
    Darum geht’s ja auch nicht. Aber wenn Theorie A 1000 Fakten erklärt, die Erklärung für Fakt 853 aber etwas aufwändig ist, während Theorie B nur Fakt 853 erklärt, dafür aber sehr elegant, und die anderen 999 ignoriert oder zu ihnen im Widerspruch steht, dann sehe ich nach wie vor keinen Grund, Theorie B und ihrer Erklärung für fakt 853 besondere Aufmerksamkeit zu schenken. Und wie gesagt, wenn du das anders siehst, steht es dir doch frei, dich damit zu beschäftigen.

    ” Welche Theorie sollten wir wählen, wenn formal und prognostisch kein Unterschied besteht? ”
    Das ist ja in deinem Beispiel Äthertheorie vs SRT definitiv nicht der Fall. Deswegen ja mein Beispiel mit der QM – da haben wir gena diesen Fall.

    “Das “dessen” bezog sich auf “Einzelfall”,”
    Dann verstehe ich den Staz imemr noch nicht – was ist denn die Algmemeinheit des Einzelfalls?

    “den “Autoritätsbeweis” zur Theoriewahl andeuten, der Doktoranden mitunter begegnet.”
    Ist das so? Klar kann man ner Doktorandin mal sagen “Glaub das jetzt einfach mal und verfolge den Beweis nicht nach, das bringt dich im Moment nicht weiter” – aber das ist nun mal meine Aufgabe als betreuerin, manchmal die Richtung vorzugeben. Mit “Autoritätsbeweis” hat das in meinen Augen nichts zu tun – genauso wie es in Ordnung ist, wenn eine Sporttrainerin einer Sportlerin sagt, auf welche Aspekte sie sich gerade konzentrieren soll. Dafür gibt es halt Betreuerinnen.

  63. #63 Benjamin Packisch
    24. August 2015

    @MartinB:
    Mir missfällt, dass du dich extrem unpräzise ausdrückst.
    “Extrem”, nun ja, ich denke nicht, dass ich hier lalle ;), aber gut, mindestens verstehst Du mich offenbar nicht, warum auch immer.
    Dass Du das Faserkreisel-Beispiel nicht kanntest, aber gleichzeitig einen quantitativen Unterschied zwischen dessen Beschreibung gegen ein (unrelatives) Ruhesystem und der ART-Beschreibung behauptest, verwundert mich, weil es mir unlogisch erscheint.
    Wie meinst Du das genau?

    Auch würde ich gerne verstehen, warum die QM ein Beispiel legitimer Deutungsdivergenz sei, die SRT aber z.B. nicht.
    Die Numerik der SRT ist ebenfalls ziemlich vielseitig, und damit sehr praktisch, aber Experimente qualifizieren Messwerte nunmal nicht, sondern sie quantifizieren sie nur. So gibt es laut einer der SRT-Testheorien (Mansouri-Sexl) logisch immer ein bevorzugtes Ruhesystem zur Beschreibung, aber demgegenüber eine m.E. erstaunliche Abneigung, SRT-Rechnungen auch so durchzuführen.
    Welchen logischen Sinn erkennst Du darin, Theoriedeutungen zu vereinheitlichen? Die QM zeigt z.B. ja, dass das nicht zwingend passieren muss. Welchen wissenschaftlichen Sinn hat Deiner Meinung nach der Streit um Theorie-Deutungen?

  64. #64 MartinB
    24. August 2015

    @Benjamin
    “aber gleichzeitig einen quantitativen Unterschied zwischen dessen Beschreibung gegen ein (unrelatives) Ruhesystem und der ART-Beschreibung behauptest,”
    Habe ich das? Dann habe ich mich falsch ausgedrückt – wo meinst du?

    “warum die QM ein Beispiel legitimer Deutungsdivergenz sei, die SRT aber z.B. nicht.”
    Weil in der QM verschiedene Deutungen dieselben Vorhersagen machen – bei der SRT gibt es das nicht; Äthertheorien machen eben nicht absolut identische Vorhersagen zur SRT.

    ” demgegenüber eine m.E. erstaunliche Abneigung, SRT-Rechnungen auch so durchzuführen.”
    Daran ist wenig erstaunlich – die Erfahrung zeigt, dass man die meisten allgemeinen Rechnung am besten in explizit kovarianter Schreibweise macht und erst am Ende konkret ein Koordinatensystem einführt. Genauso wie wir die Maxwellgleichungen auch nicht mehr komponentenweise hinschreiben, wie maxwell das noch getan hat.

    Kannst du bitte mal konkret sagen, ob du jetzt über “Alternative Interpretationen” der SRT sprichst (also solche, die identische Vorhersagen machen) oder über “alternative Theorien” (also solche, die andere Vorhersagen machen)? So ist mir das immer noch zu unscharf, um zu wissen, was du eigentlich meinst/willst. Die Mansouri-Sexl-Theorie ist ja anscheinend eine, die von der Standard-SRT abweichende Vorhersagen macht (bzw. die Parameter frei lässt, die in der SRT festliegen), allerdings bisher nicht in einem Bereich, wo man den Unterschied messen könnte. Da sie komplizierter ist, gibt es dann wenig Grund, sie zu benutzen, oder?
    Wikipedia schreibt ja auch
    “Mansouri/Sexl, und praktisch alle modernen Physiker, sind allerdings weiterhin der Meinung, dass die spezielle Relativitätstheorie und die Lorentzsymmetrie zu bevorzugen seien, da ansonsten die Äquivalenz der Inertialsysteme zerstört wäre, oder genauer gesagt, dass die beobachtete Äquivalenz ansonsten nur eine scheinbare wäre. Das macht Modelle wie die lorentzsche Äthertheorie unter Berücksichtigung von Ockhams Prinzip derart unwahrscheinlich, dass sie praktisch nicht mehr in der Fachwelt vertreten werden.”
    Also – wenn’s dir Spaß macht, kannst du die Theorie ja gern verwenden, da wo sie identische Vorhersagen macht.
    “Welchen wissenschaftlichen Sinn hat Deiner Meinung nach der Streit um Theorie-Deutungen?”
    Kann helfen, die Theorie besser zu verstehen. Wenn die Vorhersagen identisch sind, ist es ansonsten egal, ob ich die Viele-Welten-Deutung oder die Kopenhagener Deutung oder sonstwas nehme.

  65. #65 Benjamin Packisch
    24. August 2015

    @MartinB
    Zu den beiden Rechenwegen beim Sagnac-Interferometer:
    Du kanntest das Beispiel nicht, und behauptetest, beide Rechenwege ergäben unterschiedliches, und das stimmt nicht, wenn man als Maßgabe die Messwerte nimmt. Die werden durch keine der Theorien besser oder schlechter beschrieben, sondern gleich, aber mit unterschiedlichen qualitativen “Fehler”-Quellen bzw. -zuschreibungen.

    Genau das gilt analog für jede Form unterschiedlicher Beschreibungs- und Prognose-Numeriken.

    Der Punkt, den ich im Rahmen Deines Artikel hier gerne übermittelt hätte, ist der: In der Wahl der Numerik-Deutungen herrscht kein Zwang, sondern nur ein impliziter Gewohnheits-Konsens. Der Gewohnheits-Konsens kontrolliert z.B., was als kompliziert und was als einfach aufgefasst wird.
    Aus numerischer Präzision kann man logisch präzise keine qualitatitven Schlüsse oder Seins-Aussagen ziehen, und dennoch passiert es am laufenden Meter.

    Entweder macht macht man rein deskriptive Numerik und enthält sich jeder Deutung (wie in der QM), oder man stellt qualitativen Axiome auf und bearbeitet die Numerik solange, bis sie das Gewohnte oder Geforderte ergibt (wie in den RTen in der Wahl Koordinateensystems bei expliziten Messergebnisvorhersagen).

    Beides zu beanspruchen, wie es die Physik üblicherweise und Du für Sie im Artikel indirekt tust, indem Du Physik ohne Mathematik als irrelevant darlegst, heißt: Die Rechnung ohne den erkenntnistheoretischen Wirt zu machen.

    Wer oder was genau dieser “Wirt” ist, würde eine genauere Unterredung lohnen. Ich habe das hier durch Beispiele versucht anzudeuten, will Deine Zeit und den Raum aber nicht über Gebühr beanspruchen.

    P.S.: Der zitierte Aspekt von Mansouri-Sexl bezieht sich nicht auf die SRT-Numerik oder freie Parameter, sondern auf die Deutung der SRT-Numerik, denn zahlenmäßig eine Testtheorie mit abweichenden Prognosen ergäbe durchaus keinen Sinn.

  66. #66 MartinB
    24. August 2015

    @Benjamin
    “behauptetest, beide Rechenwege ergäben unterschiedliches”
    Ich sehe nach wie vor nicht, wo ich das gesagt habe – ich habe doch geschrieben
    “es ist also nicht überraschend, dass man auch mal eine falsche Theorie findet, die eine richtige Vorhersage macht, und zwar sogar einfacher.”

    “Die werden durch keine der Theorien besser oder schlechter beschrieben, ”
    Welche Messwerte? Alle, die mit der SRT beschrieben werden? Oder nur die, die sich auf diesen Spezialfall bezogen?

    Sorry, ich verstehe immer weniger, was du zu sagen versuchst. Die letzten Absätze habe ich auch wieder gar nicht verstanden.

  67. #67 Benjamin Packisch
    24. August 2015

    @MartinB
    Ich sehe nach wie vor nicht, wo ich das gesagt habe – ich habe doch geschrieben
    “es ist also nicht überraschend, dass man auch mal eine falsche Theorie findet, die eine richtige Vorhersage macht, und zwar sogar einfacher.”

    Ja, aber entscheidend ist, dass es sich bei der einfacherern Vorhersage um eine Ätherauffassung handelt. Wenn ich das auf Deine Aussage beziehe:

    ” Welche Theorie sollten wir wählen, wenn formal und prognostisch kein Unterschied besteht? ”
    Das ist ja in deinem Beispiel Äthertheorie vs SRT definitiv nicht der Fall.

    dann liegt da ein Widerspruch vor.
    Mansouri-Sexl ist in dem Zusammenhang relevant, weil die zeigen, dass man ALLE SRT-Effekte mit einem eindeutigen, ruhenden Bezugssystem rechnen kann. Beim Sagnac-Interferometer ist das etwas verwickelter in der üblicheen ART-Rechnung durch die Drehung, aber es ist grundsätzlich ein weiterer Beleg für die Deutungsfreiheit auch bei relativistischen Effekten.

    Aber ich will Dich nicht weiter mit diesen fiesen erkenntnistheoretischen Drachen belästigen, denn sie sprengen womöglich wirklich den Anspruch Deines Blogs. Danke für Deine Antworten trotzdem.

  68. #68 MartinB
    24. August 2015

    @Benjamin
    Ich bin nach wie vor verwirrt. Redest du von einer Äthertheorie, die *bezüglich aller Experimente* dieselben Vorhersagen macht, wie die SRT (und für den einen Fall besonders einfach ist), oder von einer, die dies nur in dem einen von dir zitierten Fall tut?
    Falls letzteres (so hatte ich dich bisher immer verstanden), dann habe ich dazu alles gesagt, falls ersteres – je, dann gibt es einen Fall, wo die Äthertheorie besonders elegant ist, und eben dutzende andere, wo sie es nicht ist. Es ist dann leztlich Geschmackssache und – wie gesagt – niemand hindert dich dran, alle Rechnungen mit der Theorie zu machen.

    ” diesen fiesen erkenntnistheoretischen Drachen belästigen, denn sie sprengen womöglich wirklich den Anspruch Deines Blogs.”
    Zum Abschluss ein bissschen passiv-aggressiv kommt immer gut…

  69. #69 Benjamin Packisch
    24. August 2015

    @MartinB
    Ja, also ich habe alles gesagt, was mir sinnvoll erscheint. Mein Punkt war der, dass formale Beschreibungen allein keine eindeutige Physik erlauben, dass es dafür diverse Beispiele gibt, und dass deshalb Deine Aussagen im Artikel m.E. etwas für den physikalischen Dissidenten Wesentliches auslassen.
    Jetzt stehts hier in den Kommentaren, und wer immer das liest o.ä., möge was damit anfangen, oder nicht.

    ” diesen fiesen erkenntnistheoretischen Drachen belästigen, denn sie sprengen womöglich wirklich den Anspruch Deines Blogs.”
    Zum Abschluss ein bissschen passiv-aggressiv kommt immer gut…

    Meinst Du Dich oder mich ;-)?

  70. #70 Niels
    24. August 2015

    @Benjamin Packisch

    Mein Punkt war der, dass formale Beschreibungen allein keine eindeutige Physik erlauben, dass es dafür diverse Beispiele gibt

    Wie wärs denn dann mit einem der vielen anderen Beispiele anstatt des Sagnac-Interferometers?

    Welche Theorie sollten wir wählen, wenn formal und prognostisch kein Unterschied besteht? Die einfachste? Die anschaulichste? Die mathematischste? Die umfassendste? Die üblichste?

    Die einfachste.
    https://de.wikipedia.org/wiki/Ockhams_Rasiermesser
    Vereinfacht ausgedrückt besagt es:
    1. Von mehreren möglichen Erklärungen desselben Sachverhalts ist die einfachste Theorie allen anderen vorzuziehen.
    2. Eine Theorie ist einfach, wenn sie möglichst wenige Variablen und Hypothesen enthält, die in klaren logischen Beziehungen zueinander stehen, aus denen der zu erklärende Sachverhalt logisch folgt.

  71. #71 Benjamin Packisch
    25. August 2015

    @Niels
    Also ich werde jetzt nicht alles nochmal schreiben, was ich MartinB schon übermittelt habe. Siehe Post #65 für die knackigste Darstellung. Zur Logik des Einwands frag bitte spezifisch.

    Zum Ockham-Messer nur soviel, dass es ohne überempirische adhoc-Annahmen nicht funktioniert. Ein Beispiel: Die Annahme von Kräftefreiheit, wenn sich eine Größe nicht ändert. Kräftezerlegung ist generell nicht eindeutig, so dass jede Aufteilung anwendungsspezifisch erfolgen muss. Kräftefreiheit wäre z.B. bei stationären Bedingungen der einfachste Ansatz, aber im Allgemeinen falsch.

    Zu mehr Beispielen, bei denen eine qualitative Vieldeutigkeit besteht, neben dem Faserkreisel aus der Luftfahrt, der ein einwandfreies Beispiel dafür ist, eine Auswahl:

    1. Die Aufteilung der Beschleunigungsterme in den Euler- bzw. Navier-Stokes-Gleichungen. Empirisch messbar ist nur die Geschwindigkeit und die Beschleunigung an einem Ort, die Gleichungen selbst aber enthalten eine jeweils den Mess-, Eingriffs- und Vorstellungsmöglichkeiten entsprechende Aufteilung. Zum Beispiel ist die Annahme eines Druckes in einem inkompressiblen Fluid grundsätzlich unphysikalisch, aber praktisch für die Rechnung. Formal: Du/Dt = “Summe wilkürlich aufgeteilter Beschleunigungsterme”.
    Verwandt damit gibt es erstaunliche Arbeiten von Otto Wiener, v.a. zu den relativ unbekannten Subtilitäten beim Versuch, variable Geschwindigkeitsdifferenzen zweier Objekte von einem dritten aus zu beschreiben.

    2. Die Freiheit der Koordinatensystemswahl in RT-Betrachtungsweise: Es gibt für jeden physikalischen Effekt soviele Numeriken, wie sich Inertialsysteme ausdenken kann, und für jede Numerik kann man eigene qualitative Deutungen “erfinden”. Ein m.E. lustiges und extremes, aber zugegebenermaßen vielleicht zufälliges Beispiel ist die Berechnung der Photon-Wellenlänge bei der Thomson-Streuung mit akustischem Doppler-Effekt: Es kommt exakt das gleiche heraus wie in der (ultra(!)-) relativistischen Rechnung.

    3. Ein ganze Reihe Vieldeutigkeiten bestehen bei der Berechnung von atomaren Spektrallinien. Man schaue sich nur die Bohr’sche Numerik an, und dagegen die “Black-Box”-Methodik der QM, mit dem Metaphysik-König der theoretischen Physik, nämlich dem Wahrscheinlichkeits-“FLUID” ;-).

    Das soll erstmal genügen
    Grüße
    BP

  72. #72 Niels
    25. August 2015

    @Benjamin Packisch
    Mit #65 kann ich wenig anfangen, mit deinen sonstigen Darstellungen leider auch nicht besonders viel. Ist alles irgendwie sehr vage und unkonkret. Deswegen auch die Frage nach weiteren Beispielen.

    Zu den Beispielen:

    1. Versteh ich nicht. Man kann die Navier-Stokes-Gleichungen in bestimmten Fällen durch die Euler-Gleichungen approximieren. Oder eben durch die Annahme eines inkompressiblen Fluids zu einer manchmal genügend guten Näherungslösung kommen.
    Na und?

    2. Dass man in der RT für jeden beliebigen Beobachter das richtige Ergebnis errechnen kann (“Koordinatensystemswahl”), soll ein Problem sein? Warum?
    Die Sache mit der Thomson-Streuung ist dann genau der selbe Fall wie beim Sagnac-Interferometer.
    Dazu hat MartinB schon alles gesagt:
    “es ist also nicht überraschend, dass man auch mal eine falsche Theorie findet, die eine richtige Vorhersage macht, und zwar sogar einfacher.”
    Wohlgemerkt: Eine einzige richtige Vorhersage in einem bestimmten Spezialfall. Selbstverständlich ist das Zufall, was sollte es denn sonst sein?
    3. Dieses Beispiel ist noch unsinniger. Das Bohrsche Atommodell macht bei grobem Hinsehen in einem einzigen Fall recht brauchbare Vorhersagen, aber schon bei genauerer Untersuchung dieses Falles versagt es völlig (Lamb-Shift, Hyperfeinstruktur).
    Der “Metaphysik-König” erklärt die Beobachtungen dagegen beim Wasserstoffatom perfekt. Und eben nicht nur beim Wasserstoffatom.

  73. #73 Benjamin Packisch
    25. August 2015

    @Niels
    …uff, ich glaube wir drehen uns hier im Kreis, und so ein Meinungsschlagabtausch hilft nun niemandem weiter.
    Ich glaube ehrlich gesagt auch, dass Du Modell und Realität verwechselst oder verwechseln willst, und Du deswegen die Beispiele ablehnst.
    Aber gut, meinetwegen nochmal:
    1. Die E- und NS-Gleichung sind Ausdruck geschickter mathematischer Intuition, an der man sehr schön sehen kann, dass physikalische Theorien nicht vom Himmel fallen, sondern an die menschlichen Bedürfnisse angepasst sind. Kurz: Die Theorien sind nicht besser oder schlechter als unsere praktisch orientierten und letztlich willkürlichen Annahmen. Wer das sehen will, kann es an dem Beispiel leicht sehen.

    2. Die numerische Unbestimmtheit der SRT ist kein Problem, sondern ein außerordentlich praktischer Freiheitsgrad. Und der verdient Erwähnung, weil sonst leicht der (falsche) Eindruck entsteht, man hätte es wie in der Poliitk mit “Alternativlosigkeit” der Anschauung zu tun. Das S-Interferometer z.B. hätte ich gerne schon in meinem Studium kennengelernt, einfach weil es in der Luftfahrt so viel verwendet wird, und eigentlich sehr simpel ist.

    3. “Noch unsinniger”, nun ja … denn so richtig viel mehr als das Wasserstoffatom kann man ohne weitere Annahmen nicht mit dem Wahrscheinlichkeitsfluid berechnen – auch wenn das ziemlich genau geht.
    Für komplexere Moleküle muss man Zusatz-Annahmen treffen, die letztlich ein empirisches Fitting sind – warum auch nicht, denn es ist qualitativ das gleiche Vorgehen wie beim Bohr-Modell. Aber es ist kein außerordentliches Qualitätsmerkmal einer Theorie.

  74. #74 MartinB
    25. August 2015

    @Benjamin
    “so richtig viel mehr als das Wasserstoffatom kann man ohne weitere Annahmen nicht mit dem Wahrscheinlichkeitsfluid berechnen – auch wenn das ziemlich genau geht.”
    Das hast du hoffentlich nicht ernst gemeint. Man kann mit der Qm nichts außer dem H-Atom berechnen? Schon mal was von Bandstrukturen in Festkörpern gehört? Oder Elektronenstreuung? Oder Aharonov-Bohm-Effekt? Halbleiter? Legierungsdesign (ja, das machen wir inzwischen auch mit QM, das ist ziemlich cool…)?

  75. #75 Dr. Webbaer
    25. August 2015

    Zum Abschluss noch etwas in eigener Sache: Nein, typischerweise habe ich wirklich keine Zeit und keine Lust, mir eure Privattheorie über die Physik anzuhören – wie gesagt, ihr seid nicht allein, ich habe nen ganze Haufen mails (und Blog-Kommentare) von Leuten, die die Physik revolutionieren wollen und sich für eine zweite Galilei halten.

    Nette Idee, lieber Hr. Dr. Bäker, aber lassen’S die Leutz dann aber auch ausreden.
    So ein Beitragsstrang für abweichende Meinung zur Veranstaltung der Physiklehre macht Sinn, er sammelt dann auch eher abwegige Sicht.
    Dr. W wird ein Auge drauf haben….
    MFG
    Dr. W (der nicht i.p. Physiklehre grundsätzlich abweicht, sich nur im Erkenntnistheoretischen und Philosophischen ein wenig bemüht, wenn überhaupt, vielleicht gelegentlich auch im Politischen)

  76. #76 Dr. Webbaer
    25. August 2015

    PS:
    Es haben sich ja schon, sozusagen zuckernetzmäßig angezogen, einige kluge Esoteriker anziehen lassen, also Bündelungseffekte und so haben sich bereits ergeben.
    Herr *Es geht in der Wissenschaft nicht [um] Mehrheit, sondern um Wahrheit* Packisch hat insofern schon Nahrung und Fuß gefasst.
    ‘Wahrheit’ erlaubt sich der Schreiber dieser Zeilen denn auch gleich anzumerken, gibt es in tautologischen Systemen, wie bspw. in der Mathematik oder der Philosophie oder in dafür bereit stehender Formalwissenschaft, wenn der Eigenschaftswert ‘wahr’ dort bereit steht.

  77. #77 Niels
    25. August 2015

    @Benjamin Packisch

    Ich glaube ehrlich gesagt auch, dass Du Modell und Realität verwechselst oder verwechseln willst, und Du deswegen die Beispiele ablehnst.

    Du hast behauptet, dass
    formale Beschreibungen allein keine eindeutige Physik erlauben, dass es dafür diverse Beispiele gibt.

    Deine “Beispiele” zeigen dies allerdings nicht.

    1.

    Die E- und NS-Gleichung sind Ausdruck geschickter mathematischer Intuition, an der man sehr schön sehen kann, dass physikalische Theorien nicht vom Himmel fallen, sondern an die menschlichen Bedürfnisse angepasst sind. […]
    Wer das sehen will, kann es an dem Beispiel leicht sehen.

    1. hat also überhaupt nichts mit der “Eindeutigkeit der Physik” zu tun sondern ist ein Beispiel für etwas völlig anderes?

    Die Navier-Stokes-Gleichungen kann man übrigens aus Grundtatsachen wie Masse-, Energie- und Impulserhaltung herleiten. Das würde ich nicht unbedingt “letztlich willkürliche Annahmen” nennen.

    2.

    Die numerische Unbestimmtheit der SRT ist kein Problem, sondern ein außerordentlich praktischer Freiheitsgrad. Und der verdient Erwähnung, weil sonst leicht der (falsche) Eindruck entsteht, man hätte es wie in der Poliitk mit “Alternativlosigkeit” der Anschauung zu tun.

    Das Relativitätsprinzip ist doch ganz bestimmt nicht etwas, das in der RT nirgends erwähnt wird?
    Außerdem beschreibt dieses Prinzip natürlich keine “numerische Unbestimmtheit” und selbstverständlich ist die RT “alternativlos” in ihren Vorhersagen. Da gibt es keine Beliebigkeit oder “Freiheitsgrade”.

    Das ist aber eigentlich völlig unerheblich.
    Wie gesagt, wenn die “Photon-Wellenlänge bei der Thomson-Streuung” berechnet mit dem akustischem Doppler-Effekt oder der Sagnac-Effekt über die Annahme eines ruhende Äthers das richtige Ergebnis ergibt, dann zeigt das nur, dass bei der unglaublichen Vielzahl physikalischer Vorgänge eben auch ab und zu das richtige herauskommt, wenn man einfach mal willkürlich Formeln, in denen passende Größen auftauchen, ineinander einsetzt.
    Deswegen hat man aber doch selbstverständlich keine widerspruchsfreie physikalischen Theorien gefunden. Schließlich werden die zugrunde liegenden Grundannahmen ja mit jedem beliebigen anderen Experiment falsifiziert.
    Du hast das ja selbst in #59 ja schon “Koinzidienz” genannt und mehr ist es eben auch nicht.
    Wenn man genügend viele falsche Ansätze auf ausreichend viele verschiedene Grundbedingungen anwendet, kommt für spezielle Kombinationen eben zwangsläufig auch mal eine richtige Formel heraus.

    Das du dich derart schwer mit Beispielen tust, zeigt aber schon, dass das nicht gerade häufig der Fall ist.

    3.

    Für komplexere Moleküle muss man Zusatz-Annahmen treffen, die letztlich ein empirisches Fitting sind – warum auch nicht, denn es ist qualitativ das gleiche Vorgehen wie beim Bohr-Modell.

    Sorry, aber das ist ganz großer Quark. Das Standardmodell der Teilchenphysik (bzw. die in ihm zusammengefassten QFTs) ist eine echte grundlegende physikalische Theorie.
    Als solche kann sie aus einigen wenigen Grundannahmen alle quantenmechanischen Phänomene beschreiben.
    Dass man da andauernd “empirisch fitten” muss ist absoluter Unsinn.

    ich glaube wir drehen uns hier im Kreis, und so ein Meinungsschlagabtausch hilft nun niemandem weiter.

    Na ja, der “Meinungsschlagabtausch” beruht darauf, dass du behauptest hast, es gebe einen deutlichen Wertungs-”Bias” zugunsten der Mainstream-Lehrmeinung.
    Bis jetzt konntest du noch nicht belegen, dass das nicht einfach nur daran liegt, dass es schlicht keine funktionierenden Alternativen zu den großen physikalischen Mainstreamtheorien gibt, also gar kein “Bias” vorliegt.
    Aber das hat MartinB schließlich schon vor geraumer Zeit ausgeführt:

    Aber wenn Theorie A 1000 Fakten erklärt, die Erklärung für Fakt 853 aber etwas aufwändig ist, während Theorie B nur Fakt 853 erklärt, dafür aber sehr elegant, und die anderen 999 ignoriert oder zu ihnen im Widerspruch steht, dann sehe ich nach wie vor keinen Grund, Theorie B und ihrer Erklärung für fakt 853 besondere Aufmerksamkeit zu schenken

    Wobei B der physikalischen Definition nach aber eigentlich nicht einmal mehr eine Theorie ist.

  78. #78 Benjamin Packisch
    27. August 2015

    @MartinB & Niels:
    Ich will mal respektieren, dass Ihr offensichtlich und vehement anderer Meinung seid, muss aber klarstellen, dass Ihr Eure Wertungen bisher ebenfalls nur durch Schlagworte und mit Verweisen auf bestehende Üblichkeiten “belegt” habt, wobei Ihr unterstellt habt, ich würde die nicht kennen, im Gegensatz zu “Euch” and allen “amtlichen Physikern”- also naja…

    Zur Logik dieser Linie lasse ich aber gern erstmal @Dr. Webbaer. zu Wort kommen, auf dass er mir dabei vielleicht auch erkläre, inwiefern er “wahr” als “Eigenschaftswert” auffasst und die Suche Wahrheit als “esoterisch”. Besonders würde mich auch seine Meinung zur Rolle der Rhetorik beim Umgang mit Deutungsvieldeutigkeiten formaler Physik-Systeme interessieren.

    Konkret aber nochmal zur QM, und dort spezifisch zum Verhältnis von Zusatzannahmen und Theoriegehalt. Martin nannte ja einige Beispiele aus Vielteilchensystemen. Mich würde nun z.B. interessieren, wie MartinB sich vorstellt, ohne Zusatzannahmen, d.h. rein aus einer Vieltelichen-Wellenfunktion eines Atoms z.B. die Dichte oder das magnetische Moment von Kupfer vorherzusagen. Dass man das numerisch so drehen kann, dass es hinreichend stimmt, steht außer Frage. Die Frage ist, wieviele (gutwillige!) Annahmen man für welche Genauigkeit braucht.

    Zu den SRT-Systemen nochmal zur Klarstellung: Dort hat eine Größe A genauso viele konkrete numerische Werte Ai, wie ich Bezugssysteme definieren kann. Das meinte ich mit numerischer Unbestimmtheit. Der Bezug zwischen A und anderen Größen Bj im gleichen System mag lorentz-invariant sein, etwa durch die Maxwell-Gleichungen beschrieben für A=E-Feld und B B-Feld, oder ganz einfach die Relativgeschwindigkeit Dv = vi-vj zweier Massenpunkte i und j.
    Aber für jede praktische Messung muss man sich dennoch festlegen, wogegen man relativ misst, weil alles andere undeutbar wäre.

  79. #79 MartinB
    27. August 2015

    @Benjamin
    ” d.h. rein aus einer Vieltelichen-Wellenfunktion eines Atoms z.B. die Dichte oder das magnetische Moment von Kupfer vorherzusagen.”
    Dazu gibt es diverse Möglichkeiten – im Ashcroft/Mermin z.B. steht, wie man Bandstrukturen berechnen kann, heutzutage macht man sowas mit Dichtefunktionaltheorie (bei der allerdings tatsächlich ein paar Extra-Annahmen eingehen).
    Aber deine Aussage war ja, ohne Zusatzannahmen könne man *nur* das H-Atom berechnen. Das ist definitiv falsch – man kann z.B. Metalle im Sommerfeld/Bloch-Modell beschreiben und eine ganze Menge herausbekommen, ohne dass man dazu wesentlich mehr bräuchte als die SGL (steht auch im Ashcroft).
    Dass man manche Dinge nur numerisch ausrechnen kann, heißt darüber hinaus ja nicht, dass da echte Zusatzannahmen eingehen. Numerik ist eine exakte Disziplin – da werden nicht irgendwelche Zusatzannahmen gemacht und irgendwas hingedreht, sonst hätten die Berechnungen ja keine Vorhersagekraft. Falls du anderer Ansicht bist, bitte am konkreten Beispiel.

    Und ein anderes schlagendes Beispiel für die verblüffende Vorhersagekraft der SGL hatte ich ja auch mit dem AB-Effekt angeführt.

  80. #80 Benjamin Packisch
    27. August 2015

    @MartinB:
    Wunderbar, Du sagtst es: “Dazu gibt es diverse Möglichkeiten”, Und mit einem µ guten Willen, könntest Du jetzt zugeben, dass man eben auch hier zumindest ein paar Zusatzannahmen braucht, die über die Theorie hinausgehen.
    Im übrigen kann man aber leider sogar das H-Atom ohne nicht Zusatzannahmen berechnen: Man mussannehmen, dass gebundene Elektronen im Kernfeld nicht strahlen, und das ist leider keine Kleinigkeit.
    Das darf allerdings nicht heißen, dass die SGL kein Geniestreich wäre oder gar unbrauchbar – nur ist sie eben weniger vollständig als es gerne behauptet wird von Leuten, die ihre Zunft bedroht fühlen.

    Und ja, Numerik ist exakt, aber nur formal und physikalisch nur unter einer Voraussetzung: Es darf nur plausibles, d.h. Erwartbares und Erwartetes passieren. Mathematisch kann man das astrein darstellen, aber nicht physikalisch, weil man dort nicht mehr weiß als die Messung und Erfahrung aus anderen Messungen. Man kann letztlich nur hoffen, dass eine Numerik etwas sinnvolles produziert.
    Bestes Beispiel ist die CFD, bei der man für die Navier-Stokes-Gleichung nicht mal beweisen kann, dass immer eine Lösung existiert, geschweige, das sie eindeutig wäre. Trotzdem kann man damit wunderbar arbeiten, wenn man x Zusatzannahmen (“Numeriken”) reinsteckt

    Der AB-Effekt ist tatsächlich eine sehr schöne Sache – dachte ich anfangs. Aber auch hier plagt ehrlicherweise die Frage, welche Rolle die Strahlung beim Durchgang durch den Magnetring spielt. Einen Elektrotechniker konnte ich jedenfalls nicht von der physikalischen Existenz eines Vektorpotentials überzeugen, er meint einfach, logisch, ist Strahlungsenergieverlust der die e-Phasendifferenz bewirkt. Einwände?

  81. #81 MartinB
    27. August 2015

    @Benjamin
    “Man mussannehmen, dass gebundene Elektronen im Kernfeld nicht strahlen, und das ist leider keine Kleinigkeit.”
    Äh, nein, das folgt unmittelbar aus der SGL, weil die Energieniveaus von unten begrenzt sind. Das war doch eine der großen Leistungen der QM gegenüber dem Bohr-Modell, dass diese ad-hoc-Annahme von Bohr aus der Gleichung folgt.

    “die ihre Zunft bedroht fühlen. ”
    Kannst du bitte mal diese – zur Sache nichts beitragenden – wertenden Hineininterpretationen von Motiven unterlassen? Die nerven wirklich.

    “Numerik ist exakt, aber nur formal und physikalisch nur unter einer Voraussetzung: Es darf nur plausibles, d.h. Erwartbares und Erwartetes passieren”
    Kannst du das mal konkret am Beispiel QM darlegen: Wo siehst du z.B. bei der Hartree-Fock-Berechnung von Atomen genau die Probleme? Wo bei der Sommelfeldschen Erklärung für die Wärmekapazität von Metallen?

    “Aber auch hier plagt ehrlicherweise die Frage, welche Rolle die Strahlung beim Durchgang durch den Magnetring spielt. ”
    Welche Frage soll das genau sein? In einem konstanten B-Feld gibt es keine Strahlung.

    “inen Elektrotechniker konnte ich jedenfalls nicht von der physikalischen Existenz eines Vektorpotentials ”
    Ja, und vermutlich auch keine Klavierstimmerin. Du solltest auch mich nicht unbedingt fragen, wenn Du Ideen zur mittelalterlichen Ikonographie hast, ob die plausibel sind.

    “Strahlungsenergieverlust der die e-Phasendifferenz bewirkt.”
    Ja, und warum soll der Strahlungsverlust davon abhängen, was an einem Punkt passiert, an dem das Elektron sich gar nicht befindet?

  82. #82 Benjamin Packisch
    28. August 2015

    @MartinB.
    Zur SGL und den nicht-strahlenden e–Niveaus:
    Das mag aus der SGL folgen, aber mit welchem Argument? Weil die Gleichung so ist? Weil die Gleichung sonst gut stimmt? Man muss eben doch annehmen, dass für diesen Fall sich Elektronen anders verhalten, als üblicherweise. Wenn man dann einen strahlenden Übergang hat, stellt sich die Frage, warum die die e-Wolke dann strahlt und sonst nicht – formal ist das kein Problem, zugegeben, aber physikalisch liegt diese Frage auf der Hand. Man kann die Strahlung quantifizieren, aber nicht qualifizieren mit der SG, und das führt je nach Erklärungsanspruch zu Widersprüchen. Die kann man sehr gut pragmatisch ignorieren, aber streng genommen bleiben sie bestehen.

    Zur Numerik:
    Das Navier-Stokes-Beispiel ist okay? Oder hast Du da Einwände?
    Ein analoges Beispiel aus der QM ist schwieriger zu konkretisieren, weil dort die Gleichungen wie oben dargelegt schon weitreichende Annahmen enthalten. Letztlich ist die Wellenfunktion der SGL schon eine unphysikalische Annahme, die sich aber immer genau so erweitern lässt, wie man es gerade braucht, siehe z.B. die Erweiterung zur Dirac-Gleichung auf vier Dimensionen. Auch das ist kein Problem im pragmatischen Sinne, aber es kann dazu führen, neue Perspektiven zu verdecken.

    Zum Aharonov-Bohm-Effekt:
    Na das Feld des Elektrons ist schon ausgedehnt, und QM-technisch ist auch das Elektron selbst (was immer es “ist”) ausgedehnt. Das heißt, dass sein E-Feld beim Durchgang durch den Magnetring auch dort ungleich null ist, wo B!=0, und zwar zeitlich variabel. Dadurch tritt Induktion und damit Strahlung auf, die ihre Energie irgendwoher bekommen muss – z.B. vom Elektron.
    M.E. ist der HF-Techniker-Einwand nicht ganz doof, oder?

    P.S.: Nur für das Protokoll:
    Sehr gerne lasse ich alle Motiv-Deutungen weg, wenn Du dafür alle diffusen Inkompetenz-Unterstellungen sein lässt.
    Es geht mir auch nicht darum, die Leistungen der Physik kleinzureden, sondern darum, die m.E. oft unterschätzte Größe der Spielräume in bestehenden Anschauungen zu betonen. Die Physik ist m.E. keineswegs ein Korsett an erprobten Unmöglichkeiten, sondern eine Sammlung erprobter Möglichkeiten, die alles andere als erschöpfend sind.

  83. #83 MartinB
    28. August 2015

    @Benjamin
    “Das mag aus der SGL folgen, aber mit welchem Argument? ”
    Aus der SGL folgt für ein Elektron im Coulomb-Potential, dass die Energieniveaus von unten begrenzt sind. Es gibt also einen Grundzustand. Wegen der Energieerhaltung kann ein Elektron im grundzustand deshalb keine Energie in Form von Strahlung abstrahlen.

    Da braucht es wirklich keine Extra-annahmen oder sonst etwas (außer der Energieerhaltung), es ist wirklich so einfach.

    @Benjamin
    “Das Navier-Stokes-Beispiel ist okay? Oder hast Du da Einwände?”
    Ich habe 1. schlicht keine Lust, ein neues Fass aufzumachen und bin 2. nicht so die Expertin für die NS-Gleichungen. Ob das Argument o.k. ist, lasse ich deswegen erstmal offen.

    “QM-technisch ist auch das Elektron selbst (was immer es “ist”) ausgedehnt”
    Nein, du bekommst den Effekt, egal wie klein (oder verschwindend) die Wellenfunktion des Elektrons am Ort des B-Felds ist.

    “die ihre Energie irgendwoher bekommen muss – z.B. vom Elektron.”
    Beim AB-Effekt ändert das Elektron aber seine Energie doch gar nicht, sondern nur seine Phase. Da entsteht doch gar keine Strahlung.

    “Es geht mir auch nicht darum”
    Ehrlich gesagt ist mir inzwischen relativ egal, worum es dir geht. Deine Behauptungen sind oft diffus und teilweise mehr als abstrus (und ändern sich anscheinend auch gelegentlich…) – da ist es schnuppe welche Motivation dahinter steht. Mir geht es hier nur noch darum, keinen Unsinn unkommentiert stehen zu lassen, das ist alles.

  84. #84 Benjamin Packisch
    28. August 2015

    @MartinB:
    “keinen Unsinn unkommentiert stehen zu lassen”
    Das ist der Punkt, wie bei jeder wissenschaftlichen Kommunikation.

    zu SGL “Es ist einfach so”
    So ist es – in der Gleichung. Und dieses “Es ist so” ein reales “Amen und aus” bedeutet steht zur Debatte. Wenn Du die Frage nicht diskutieren wilst, ok, es ist Dein Forum. Aber es ist eine offene Frage.

    Zu AB.
    Wie sorgst Du denn dafür, dass bei einem Aha.-Bohm-Effekt die Wellenfunktion verschwindend klein wird ?! Also das halte ich für experimentell unmöglich.
    Dass der Effekt nur die Phase, aber nicht die Energie betrifft ist formal korrekt, aber ich bin mir nicht sicher, ob man die Energie auch gemessen hat. Die Messungen die ich kenne, haben nur die Phase vermessen.

    Zu Numerik:
    Ok, dann lassen wir das Hier würde es weitergehen.

  85. #85 Niels
    28. August 2015

    @Benjamin Packisch
    Weia. Du bist tatsächlich studierter Physiker? Oder war das mit der “nüchternen Physikerseele” anders gemeint?

  86. #86 MartinB
    28. August 2015

    @Benjamin
    “. Und dieses “Es ist so” ein reales “Amen und aus” bedeutet steht zur Debatte. ”
    Ich verstehe nicht, was du meinst. Was genau willst du debattieren? Die Korrektheit der SGL? Welche Frage soll da genau offen sein?

    “Also das halte ich für experimentell unmöglich.”
    Wenn ich keine Abhängigkeit von der WF am Ort des Magnetfelds sehe (sprich: die Phasenverschiebung ist unabhängig davon, wie weit ich das Elektron um die Spule herumführe), dann folgt daraus doch, dass es kein Effekt der direkten WeWi zwischen WF und B-Feld sein kann.

    “ch bin mir nicht sicher, ob man die Energie auch gemessen hat.”
    Aber du behauptest, dass da Energie abgestrahlt wird. Das müsstest Du dann auch belegen (wobei sich eine Energieänderung ja auch in der Phase niederschlagen sollte, weil sich dadurch die Wellenlänge ändert, oder nicht? Müsste man doch im Interferenzbild sehen.)

  87. #87 Benjamin Packisch
    28. August 2015

    @MartinB.
    Zu SGL: Ich will darauf hinweisen, dass dort Fragen offen sind, logischer und physikalischer Art. Deine Antwort habe ich allmählich in extrapolierbarer Deutlichkeit verstanden: Du (er)kennst keine.

    Zu AB:
    Ich stelle hier die Möglichkeit der Energieabstrahlung zur Debatte beim AB-Effekt, und zwar a) weil es das gleiche Prinzip all der anderen Beispiele verdeutlicht und b) weil Du danach wiederholt gefragt hast.

    Das AB-Experiment ist extrem empfindlich, und von der Rechnung bis zur Durchführung sind nicht umsonst mehrere Jahrzehnte vergangen ((1939?)1959 -1986), aber das nur am Rande.
    Ob Wellenfunktion oder nicht: EIne bewegte E-Feldquelle in einem B-Feld strahlt, und die Reichweite des E-Feldes eines Elektrons lässt sich nicht per Dekret vermindern. Die m.E. offene Frage ist, wie die Rückwirkung auf dass die Spule durchquerende Elektron konkret aussieht.

    Fakt ist, dass der Effekt, weil er so winzig ist, nur durch Interferenz mit einem zweiten, außerhalb der Spule passierenden Elektrons nachzuweisen ist. Es wäre ein leichtes, die beobachtete Phasenverschiebung durch einen Wellenlängenveränderung zu errechnen.
    Ich bin ziemlich sicher, dass man hier numerische Ergebnis-Äquivalenz erzielen könnten zwischen klassischer und QM-Beschreibung.

    Aus der Tatsache, dass man nur eine Phasendifferenz solide beobachten kann, lässt sich eben nicht folgern, dass es sonst keine Veränderungen geben kann.

    Aber auch das verdeutlicht nur das Prinzip, dessen, was ich hier gegen doch erhebliche Widerstände zu vermitteln versuche.

  88. #88 MartinB
    28. August 2015

    @Benjamin
    “Du (er)kennst keine. ”
    Vielleicht könntest du die Fragen ja mal klar formulieren – die nach der Abstrahlung haben wir ja hoffentlich geklärt.

    “weil Du danach wiederholt gefragt hast. ”
    Nein, du hast das aufgebracht, als du deinen ETechniker zitiert hast.

    “1959 -1986″
    Äh, nein. Die ersten Messungen des AB-Effekts waren deutlich früher, das weiß sogar Wikipedia
    ” Experimentell wurde dieser Effekt Anfang der 1960er Jahre u. a. von Möllenstedt und Robert G. Chambers[7] nachgewiesen.”

    “Aus der Tatsache, dass man nur eine Phasendifferenz solide beobachten kann, lässt sich eben nicht folgern, dass es sonst keine Veränderungen geben kann”
    Neion – aber eine änderung der Energie der Elektronen müsste doch die Wellenlänge und damit das Phasenmuster ändern, oder nicht?

    “Ich bin ziemlich sicher, dass man hier numerische Ergebnis-Äquivalenz erzielen könnten zwischen klassischer und QM-Beschreibung. ”
    Dann kannst du das ja mal rechnen und veröffentlichen. Dass du dir ziemlich sicher bist, ist als Beleg etwas dünn…

  89. #89 Benjamin Packisch
    28. August 2015

    @MartinB:
    “Äh, nein. Die ersten Messungen des AB-Effekts waren deutlich früher, das weiß sogar Wikipedia”
    Aus dem Wort “sogar” schließe ich die Fruchtlosigkeit weiterer Bemühungen hier.
    Wenn dieser Benjamin sogar hohler noch als Wikipedia ist, ja, was dann eigentlich?!
    Mein Kenntnisstand ist der, entgegen der Wikipedia, dass die ersten unstrittigen Ergebnisse 1986 zustande kamen, auch wenn “nicht mal” Wikipedia das weiß ( Link)

    Wenn sich an einer Elektronenwelle Phase ODER Energie ändert, dann kann man eine Phasenänderung messen, tut mir leid, darauf muss ich bestehen.
    Ansonsten gute Nacht!

  90. #90 MartinB
    28. August 2015

    @Benjamin
    Das Experiment, das du zitierst (schöner Link, kannte ich noch nicht), schirmt die WF vollständig vom B-Feld ab. (Was du ja eben noch für nicht möglich erklärt hast.) Das Experiment, das ich zitiere, zeigt, dass der Effekt prinzipiell existiert.

    “Phase ODER Energie ändert, dann kann man eine Phasenänderung messen”
    Dann rechne doch mal vor, wie das für diese Experimente gegen soll.

    Du tust hier genau das, was ich in meinem Text thematisiere: Ohne Rechnungen qualitativ (und leider nicht mal widerspruchsfrei) argumentieren und behaupten, die Mathematik bekommt man schon irgendwie hin.

    Und nach wie vor bist du die Antwort auf die Frage schuldig, wo genau denn nun das Problem mit der SGL sein soll.

  91. #91 Benjamin Packisch
    28. August 2015

    @MartinB:
    Es kommt einer Erpressung nahe, was Du hier tust, denn auch Du gehst auf meine Argumente nur selektiv ein, und auch Du wählst Dir Aussagen aus, die Deiner offenbar vorgefertigten Meinung zu meinen Aussagen dienen. Na gut.

    Für AB beharrst Du auf der Wellenfunktion der Elektronen, und ich habe erwähnt, dass dies mit Betonung des E-Feldes der Elektronen nicht gut konform geht. Oder zaubert die WF die Ladung des Elektrons davon!?
    Zu der Berechnung, denn selber rechnen willst Du offenbar auch sehr einfach Dinge nicht:
    L sei die Wegstrecke der Welle,
    dP die zu erzielende/erklärende Verschiebung der Phase
    außerdem E = hf = hc/l (mit l der Wellenlänge)
    Gesucht ist die Änderung der Wellenlänge l2 – l (und damit ein dE), das dP hervorbringt. Dabei entspricht dP einer Weglängenänderung von l2 – l = l*dP/(2*Pi);
    ->dP ist proportional der vollen Wellenzüge modulo l, die in L passen.
    ->Nehmen wir o.B.d.A an, dass L exakt so lang ist, dass l ganzzahlig hineinpasst, also l = L/n mit n natürlich;
    -> Die Wellenlänge muss sich um dl = (l*dP/(2Pi)) / n = (l*dP/(2Pi))*l/L ändern, um den Phasenversatz dP nach dem Weg L zu produzieren.
    -> Das entspricht einer Energieänderung
    dE = hc/l – hc/(l-dl) = hc(1/l – 1/(l-(l*dP/(2Pi))*l/L))
    q.e.d.
    (und für den routinierten Physiker wirklich nur eine Fingerübung, mit Verlaub.)

    Also lassen wir es gut sein:
    Und für DIch zu einem geruhsamen Wochenende noch das geforderte Bekenntnis:

    “Die Schulphysik, die Schulphysik,
    ist frei von jeglicher Kritik!
    Wer anderes zu denken wagt,
    wird ausgesperrt, ganz unverzagt.”

    In diesem Sinne, ich habe den Wink zur Tür angenommen.
    Grüße
    BP

  92. #92 MartinB
    28. August 2015

    @Benjamin
    Ich verstehe die Rechnung ehrlich gesagt nicht, weil mir nicht klar ist, welche Strecken du da betrachtest.
    Aber dass eine Energieänderung eine Phasenverschiebung (bzw. Wellenlängenänderung) bedeutet, ist ja unbestritten – du müsstest schon vorrechnen, wie das für den AB-Effekt gehen soll.

    Aber wenn es in deinen Augen einer Erpressung gleichkommt, von Dir Belege für deine Behauptungen zu verlangen, dann lasse ich es lieber.

  93. #93 Dr. Webbaer
    29. August 2015

    Zur Logik dieser Linie lasse ich aber gern erstmal @Dr. Webbaer. zu Wort kommen, auf dass er mir dabei vielleicht auch erkläre, inwiefern er “wahr” als “Eigenschaftswert” auffasst und die Suche Wahrheit als “esoterisch”. Besonders würde mich auch seine Meinung zur Rolle der Rhetorik beim Umgang mit Deutungsvieldeutigkeiten formaler Physik-Systeme interessieren.

    1.) Der Schreiber dieser Zeilen hat sich hier keiner ‘Linie’ angeschlossen oder ausgeschlossen.
    2.) ‘Wahr’ ist ein Wert (“wie jeder andere”), er liegt im Wissenschaftlichen genau dann vor, wenn er definiert ist.
    3.) In der (mathematischen) Logik liegt er definitionsgemäß vor, wenn eine Aussage auf eine Axiomatik zurückgeführt werden kann.
    4.) In der Rechtspflege, die die Wahrheitssuche angibt zu meinen, aber mit Wahrscheinlichkeiten und Plausibilitäten arbeitet, ist etwas anderes gemeint. Dies nur stellvertretend für die weltliche Wissenschaft ergänzt, die Physiklehre ausschließend.
    5.) In der Physiklehre muss bis sollte es keinen Wahrheitsbegriff geben, es könnte alternativ und deutlich besser, wird so ja auch, von Beleglagen, von Evidenz und von der empirischen Adäquatheit von Sichten / Theorien und Sichtenbildung / Theoretisierung gesprochen und geschrieben werden.
    Naturwissenschaftlich sieht es im skeptizistischen Sinne ganz wunderbar aus zurzeit.
    6.) Die Suche nach Wahrheit ist im Naturwissenschaftlichen nicht vorgesehen, jedenfalls nicht im skeptizistichen Sinne, es wird ja seit ca. 100 Jahren nicht mehr verifiziert, sondern die Falsifikation gesucht.
    Die Suche nach Wahrheit wäre in diesem Sinne etwas für die anderen.
    7.) Rhetorik kann es auch im Naturwissenschaftlichen geben, sie ist philosophisch begründbar. [1]
    Sie muss aber keine besondere Rolle spielen und dort wo “Hardcore” stattfindet, typischerweise in der Physiklehre, Herr Dr. Bäker sei an dieser Stelle gegrüßt, darf sie sparsam verwendet werden.

    MFG + schönes WE noch,
    Dr. W

    [1]
    Die Naturwissenschaft ist eine Erkenntnislehre, sie pflegt Sichten auf Sachen oder Sachzusammenhänge in “n:m”-Beziehungen, und zwar zwischen erkennendem Subjekt und eben diesen Sachen oder Sachbezügen.
    (Natur-)Wissenschaft ist eine Veranstaltung des Primaten, insofern darf auch ein wenig rhetorisch argumentiert werden, “gemöppelt” werden, denn es geht letztlich um Erkenntnissubjekte, die Sichten pflegen, die “auch mal” persönlich angefragt werden dürfen, bspw. auch i.p. allgemeine Aufstellung und Motivationsimperativ.

  94. #94 Krypto
    29. August 2015

    @Webbaer:
    Mir ist aufgefallen, dass der Rhetorikanteil indirekt proportional zur Belastbarkeit einer “Theorie” von (Möchtegern-)Physikern ist 😀

  95. #95 Dr. Webbaer
    30. August 2015

    Dieses Interview fanden einige ganz gut:
    -> https://www.pik-potsdam.de/~stefan/mare-interview.html

  96. #96 Benjamin Packisch
    31. August 2015

    @Dr. Webbaer:
    Vielen Dank für Ihre Ausführungen.
    Sie plädieren offenbar für einen pragmatischen Wahrheitsbegriff in der Physik, also dafür, dass man “Wahrheit” nicht benötige für die Physik, sondern nur Selbstkonsistenz. Dagegen wäre m.E. nichts einzuwenden. Die Erfahrung lehrt mich aber (auch hier), dass Selbstkonsistenz häufig mit selbstbezüglicher Willkür einhergeht – und zwar auf allen Seiten.
    Kennen Sie Vaihingers “Physik des als ob”?
    Unzweifelhaft ist wissenschaftliche Begriffsbildung immer irgendwo so “als ob”, d.h. nicht aus dem “an sich” der Dinge geschöpft, sondern aus dem Willen des Wissenschaftlers. Daran ist nichts verwerfliches, solange das bewußt passiert.
    Aber verwerflich wird es, wenn für all diese willentlichen Konstruktionen vom Elektron bis zum Higgs-Boson aggressiv ein unbestreitbarer Seins-Status behauptet wird. Aus meiner Sicht ist das erkenntnistheoretisches Schindluder, und aus ihrer offenbar auch, oder?
    Polemisch könnte man die Haltung als Schützengraben-Logik bezeichnen à la:
    “Wenn hier einer grundsätzlich abweicht, dann, weil er nicht die Genialität des ‘Schützengrabens’ (=Schulphysik) ermessen kann. Wir brauchen seine Argumente nicht zu hören, weil wir ‘wissen’, dass sie für den ‘Schützengraben’ falsch sind. Lasst die Abweichler ‘vorrechnen’, dann könnt ihr sie auslachen. Und wenn nicht, dann gibt es einen wesentlichen Haken, warum wir nicht längst davon wissen.”

    Rhetorisch verbrämt liest sich das dann ähnlich wie der Artikel oben, oder wie die meisten Kommentare.
    Was m.E. helfen würde sowohl gegen Bunker-Mentalitäten und gegen die wissenschaftliche Analphabetisierung um uns: ein Rückbezug zur kopernikanischen Wende des Denkens, wie sie von Kant im eigentlichen Sinne vorgeschlagen wurde. Das wäre auch eine Richtung, mit der der Skeptizismus endlich etwas über seine Formal-Mäkeleien hinaus beitragen könnte.

    Wie sehen Sie das als Skeptiker?

    @MartinB
    Wie gesagt, es kommt einer Erpressung NAHE, wenn Du immer wieder beanspruchst, echte und falsche Kritikaspekte heraussuchen zu können, und mir Inkompetenz zu unterstellen, wenn ich nicht auf jedes Detail eingehe.
    Jetzt ist es wenigstens Konsens, dass der AB-Effekt quantitativ als Energieänderung funktionieren kann…immerhin. L war definiert als die Wegstrecke des e-
    Wellenzugs (s.o.), gemessen von einem bel. Bezug innerhalb der Kohärenzlänge aus.
    Wie das qualitativ laufen würde: Das E-Feld des Elektrons induziert eine Änderung in der Ringmagnetisierung, die wieder ein E-Feld zurückinduziert. Gemäß der Lenz’schen Regel bremst dies das e-. Auch hier kann man eine quantitative Rechnung anstellen, ich kann aber nicht garantieren, dass Du sie verstehst.

  97. #97 MartinB
    31. August 2015

    “Jetzt ist es wenigstens Konsens, dass der AB-Effekt quantitativ als Energieänderung funktionieren kann…”
    Äh, nein, da hast du mich missverstanden. Prinzipiell kann logischerweise eine Wellenlängenänderung die Phase verschieben – aber beim AB-Effekt müsstest du schon erklären, wie sich die Wellenlänge auf welchen der Wegen um die Spule herum so ändert, dass sich zwar das Muster verschiebt, der Abstand der Maxima und Minima aber derselbe bleibt. Mag sein, dass das geht, hast du aber nicht gezeigt.

  98. #98 Benjamin Packisch
    31. August 2015

    @MartinB.
    “Mag sein, dass das geht, hast du aber nicht gezeigt.”
    Äh, ich verstehe Dich nicht ganz… ich habe doch eine Erklärung abgeben. Wenn Du sie für unmöglich hälst – und zwar zunächst qualitativ – dann lass mich bitte etwas präziser wissen warum.
    Denn:
    Im Inneren der Spule wird ein e- stärker auf den Magnetfluß einwirken können als außen, weil dort das E-Feld des e- zu einem größen Anteil wechselwirken kann.
    Also, bitte etwas präziser, wenn Du das als unmöglich erklären willst.
    Dank & Gruß
    BP

  99. #99 Krypto
    31. August 2015

    @BP:

    Also, bitte etwas präziser, wenn Du das als unmöglich erklären willst.

    Frei nach dem Motto “frech kommt weiter” soll Martin Dir jetzt erklären, wieso Dein Geschwurbel keinen Sinn ergibt?
    Du stellst Behauptungen auf, dann ist es auch an Dir, sie zu belegen!

  100. #100 MartinB
    1. September 2015

    @Benjamin
    Was du gezeigt hast (wenn ich die Rechnung ohne Erklärung der Formelzeichen richtig verstanden habe): Wenn zwei Elektronen über eine Strecke L laufen und ich bei einem die Energie und damit die Compton-Wellenlänge ändere, dann kommen die beiden am Ende der Strecke mit einer Phasenverschiebung an.

    Was hätte gezeigt werden sollen:
    (1) Wie kann ich die Energie eines Elektrons, das auf zwei Wegen eine Spule umläuft, so ändern, dass am Ende die beim AB-Effekt beobachtete Phasenverschiebung herauskommt, ohne dass sich die Abstände im Interferenzmuster ändern?
    (2) Welcher Mechanismus ist für diese Energieänderung verantwortlich? Soll das eine Form von Synchrotronstrahlung o.ä. sein? In schwachen B-Feldern werden Elektronen ja normalerweise einfach auf eine Kreisbahn gebracht ohne ihre Energie signifikant zu ändern, wenn ich mich recht entsinne. Insbesondere wäre es dafür wichtig, zu überlegen, warum der Effekt auf einem der Teilwege um die Spule auftritt, aber nicht auf dem anderen – gewinnt der eine Weg Energie und der andere verliert sie? Wie genau soll der Mechanismus dahinter funktionieren? Und nein, das ist keine zusätzliche Forderung, sondern die Forderung nach einer in sich konsistenten Beschreibung, wie sie die SGL und die Standard-Erklärung des AB-Effekts liefern. (Als Bonus kannst du dir dann noch andere Phänomene mit Berry-Phase vornehmen und zeigen, dass da derselbe Mechanismus am Werk ist…)

    Und zu
    “Im Inneren der Spule wird ein e- stärker auf den Magnetfluß einwirken können als außen, weil dort das E-Feld des e- zu einem größen Anteil wechselwirken kann.”
    Hast du mir nicht gerade einen Artikel gezeigt, in dem der AB-Effekt nachgewiesen wird, ohne dass die WF einen Anteil im Inneren der Spule hat: “Evidence for teh AB-Effect was obtained with magnetic fields shielded from the electron wave”???

  101. #101 Dr. Webbaer
    1. September 2015

    @ Herr Packisch :

    Sie plädieren offenbar für einen pragmatischen Wahrheitsbegriff in der Physik, also dafür, dass man “Wahrheit” nicht benötige für die Physik, sondern nur Selbstkonsistenz. Dagegen wäre m.E. nichts einzuwenden. Die Erfahrung lehrt mich aber (auch hier), dass Selbstkonsistenz häufig mit selbstbezüglicher Willkür einhergeht – und zwar auf allen Seiten.

    Also “ein wenig mehr” als die Kohärenz von Sicht und Sichtenbildung aufrecht zu erhalten, leistet die moderne skeptizistische Wissenschaftlichkeit, die gewohnt und bereit ist mit bedarfsweise zu falsifizierenden Provisorien zu arbeiten, die auch abgelöst werden dürfen, schon.
    Sie ist nämlich der Empirie stark verbunden, sie ist zudem exoterisch, vs. esoterisch, d.h. jeder kann mitmachen, jeder könnte (mit)verstehen, und sie kanalisiert Erkenntnisprozesse, indem sie die Wissenschaftlichkeit als Sozialveranstaltung des Primaten versteht und bewirbt, so dass das allgemeine Interesse sinnhafterweise mal mehr in die eine und mal mehr in die andere Richtung geht.
    Der Nutzen, der sich über Applikationen ergibt, der werte Herr Dr. Bäker, der sich u.a. auch i.p. Materialwissenschaft bemüht, wird hier vielleicht heimlich (oder auch öffentlich [1]) ein wenig nicken wollen, ist und bleibt wichtich.

    MFG
    Dr. W

    [1]
    Vermutlich in diesem Fall eher: heimlich, der Schreiber dieser Zeilen ist hier nicht so-o gut angesehen, vielleicht auch weil er sich als Skeptizist (vs. Skeptiker) regelmäßig gegen falsch verstandenen Skeptizismus gewendet hat, gegen den Pseudoskeptizismus (das Fachwort), wobei einige dies als generelle Wissenschaftskritik verstanden, was sie auch ist, aber auch als malevolente Kritik, was sie gerade nicht ist.

    PS:
    Kant ist jetzt nicht (erneut) zu Rate gezogen worden, bei Kant ist der Schreiber dieser Zeilen auch in diesen Punkten oft “auf Distanz”, wie es so schön in der deutschen Sprache heißt.

  102. #102 Dr. Webbaer
    1. September 2015

    PS:

    Aber verwerflich wird es, wenn für all diese willentlichen Konstruktionen vom Elektron bis zum Higgs-Boson aggressiv ein unbestreitbarer Seins-Status behauptet wird. Aus meiner Sicht ist das erkenntnistheoretisches Schindluder, und aus ihrer offenbar auch, oder?

    Dezent formuliert blieben derartige Behauptungen erkenntnistheoretisch angreifbar, wie der philosophische Realismus allgemein.
    Nichtsdestotrotz können derart Postulierende ganz hervorragende Wissenschaftler sein.
    Richtig ärgerlich wird es, wenn die (Natur-)Wissenschaftlichkeit begleitenden Kräfte, also wenn keine Spezialisten, sondern medial, äh, Vermittelnde, wenn Wissenschaftskommunikatoren erkenntnistheoretisch “nichts drauf” haben, aber gegen Skeptiker und Skeptizisten wahllos möppeln… [1]

    [1]
    Der dbzgl. absolute Tiefpunkt konnte (natürlich) bei FF und Jörg Wipplinger erreicht werden.

  103. #103 Benjamin Packisch
    1. September 2015

    @MartinB:
    Ich habe noch immer den Eindruck, dass Du das Experiment zum AB-Effekt nicht 100% so verstanden hast, wie ich oder wie es in Tonomura et al. beschrieben ist.
    Bei der Art der Messung, wie sie m.E. im AB-Experiment üblich ist, DURCHquert ein Teil der e-Welle einen magnetisierten Torus, während der andere AUßen herum läuft. So ist es im Tonomura(1986)-Paper gemacht (und dort ist übrigens auch unsere Frage diskutiert, warum die Messungen von 1960 lange als zweifelhaft galten, wovon auf WP nichts steht.)
    Unsere Frage ist letztlich, ob der Magnettorus-Überzug aus Supraleiter und Cu-Beschichtung den inneren Magneten auch vollständig von E-Feld-Veränderungen im Torusdurchgang abschirmt.
    Die Formelzeichen habe ich in #91 so konventionell wie möglich gewählt (nicht explizit erklärt dort: h . Planck’sches Wirkungsquantum, c Lichtgeschwindigkeit, f Frequenz). Die Energiedifferenz wäre übrigens so winzig, dass man mit der Methode aus Tonomura et al. wohl keine Abstandsänderung der Interferenzstreifen würde sehen können.
    Okay, aber ich muss da jetzt nicht weiter darauf herumreiten, Dich überzeugt es nicht – wohingegen ich noch immer den Eindruck habe, dass Du den Punkt, auf den es mir ankommt, nicht zur Kenntnis nimmst, oder nehmen willst, weil Du ihn für falsch hälst.

    Noch einmal für wohlmeinenden Leser:
    Die Physik ändert sich NICHT, wenn ich eine andere Beschreibung wähle, es ändern sich nur Fehlerzuschreibungen und Manipulationsmöglichkeiten. Der Sinn qualitativer Variation ist der neue Blick auf Altbekanntes. Wenn man nur eine gängige formale Seite betont, übersieht man leicht, wieviel willentliche, z.T. unnötig selbstbeschränkende Konstruktion in physikalischen Theorien liegt. Ergebnisoffene Deutungsdiskussionen waren im übrigen immer Kennzeichen physikalischer Blütezeiten.

    @Dr. Webbaer, der hier offenbar auch mitunter um die Gunst des Blog-Autors versuchte zu buhlen:
    Verstehe ich sie richtig, dass Sie die Abwehr gegen “Theorie-Relativistik” (d.h. qualitative Deutungsvielfalt) ebenfalls als unlogisch erkennen? Genauer: als Funktion impliziter und/oder erzieherischer Regeln in Primaten-Sozialveranstaltungen?

  104. #104 Benjamin Packisch
    1. September 2015

    P.S: @MartinB: Noch ein Zitat aus der “Conclusion” aus Tonomura1986, dass es lohnt genau genau gelesen zu werden:
    … The most controversial in the dispute over experimental evidence of the AB effect has been whether or not the phase shift would be observed when both electron intensity and magnetic field were extremely small in the region of overlap. Since experimental realization of absolutely zero field is impossible, the continuity of physical phenomena in the transition from negligibly small field to zero field should be accepted instead of perpetual demands for the ideal; if a discontinuity there is asserted, only a futile agnosticism results. [A. Tonomura et al. PRLNo. 56-8, 1986]

  105. #105 Dr. Webbaer
    1. September 2015

    @ Herr Packisch :

    @Dr. Webbaer, der hier offenbar auch mitunter um die Gunst des Blog-Autors versuchte zu buhlen:
    Verstehe ich sie richtig, dass Sie die Abwehr gegen “Theorie-Relativistik” (d.h. qualitative Deutungsvielfalt) ebenfalls als unlogisch erkennen?

    Opa W. schön länger “im Geschäft”, aufgefallen ist ihm Dr. B, als er zusammen mit einigen intellektuell Obdachlosen seinerzeit den geschätzten WebLogger Jörg Friedrich angegangen ist, was in dessen Vertreibung mündete,
    ischt schon ca. fünf Jahre her.
    Beim als ‘Unlogisch-Erkennen’ möchten Sie vielleicht auf die Sprachlichkeit zurückführen, also, was Logik, Logos und so eigentlich sind.
    Im Moment hat Kommentatorenfreund Dr. W kein Bock drauf dbzgl. zu erläutern.

    MFG + bis denne!
    Dr. W (der sich demzufolge bis zumindest dem Fünften dieses Monats auszuklinken hat)

  106. #106 MartinB
    1. September 2015

    @Benjamin
    “Die Formelzeichen habe ich in #91 so konventionell wie möglich gewählt ”
    Du hast aber nicht erklärt, welche Länge du betrachtest – beim AB-Effekt gibt es die Weglänge einmal drumherum, ist es die? Oder eine Weghälfte? Siehe dieses Bild:
    https://de.wikipedia.org/wiki/Aharonov-Bohm-Effekt#/media/File:Aharonov-bohm.png
    Die Phasenevrschiebung zwischen was betrachtest du – zwischen den beiden Wegen rechts und links um die Spule herum? Du schreibst nur lakonisch
    “L sei die Wegstrecke der Welle,
    dP die zu erzielende/erklärende Verschiebung der Phase”
    Aber welche Welle? Auf welchem Weg? Welche Phasenverschiebung relativ wozu? Bekommen die beiden Teilde der WF auf den beiden Seiten der Spule unterschiedliche Phasenverschiebungen? Wenn ja, wie funktioniert das mit der Energieänderung?

    Was den AB-Effekt angeht – das Zitat, das du bringst, bringt es ja sehr schön auf den punkt: Ja, absolutes nicht-überlappen der Felder ist unmöglich. ABER: Der Effekt ist ja anscheinend *unabhängig* von der Größe dieses Überlapps – du müsstest also erklären, wie die Wechselwirkung zwischen Elektron und B-Feld so abläuft, dass immer der durch die SGl mit Vektorpotential vorhergesagte Effekt herauskommt, unabhängig davon, wie weit entfernt das Elektron nun um die Spule läuft und ob man da mit Supraleitern abschirmt oder nicht. Und da reicht ein “Prinzipiell kann man Phasenverschiebungen durch Energieänderungen erklären und ich kann das ganz bestimmt auch ausrechnen” nicht aus.

    Wenn du quantitativ zeigen kannst, dass man die verschiedenen Experimente zum AB-Effekt ohne Wewi mit dem Vektorpotential und über eine direkte Wewi der WF mit dem Magnetfeld richtig berechnen kann, ist das sicher eine nette Veröffentlichung wert.

    PS: Nur zur Info, da dir das anscheinend Gedanken macht: nein, ich lese die Kommentare des Webbären nicht und antworte deshalb auch nicht auf sie.

  107. #107 Benjamin Packisch
    1. September 2015

    @MartinB.
    Die Frage ist also, wie ein Vektorpotential aussehen kann, dessen Rotation Null ist. Tonomura lösen das durch einen quasi-ideal(!) abgeschlossenen ringförmigen Magnetfluss. Das WP-Bildchen kommt mir da ehrlich gesagt etwas zu ideal vor. Ob das überhaupt so möglich ist, würde erfordern, dass rot A lokal != 0 ist und außenrum = 0 UND A(Pfad A) != A(Pfad B). Ich habeZweifel, ob das so geht, wie bei WP gezeichnet.

    Ich denke, hier wäre es am besten, direkt ein separates Experiment anzuschauen, bei der ideale WP-Aufbau realisiert – allein, mir ist keins bekannt, und ich denke auch, dass diese eine ziemlich schwierige Aufgabe ist. Aber ich lasse mich gern eines besseren belehren.

    Abgesehen davon: L ist nur relevant als linearer Faktor für den Gangunterschied der beiden Wellenzüge, er geht nur in einen der beiden Pfade ein, nicht in beide – daher ist er nicht kriegsentscheidend.
    Publizieren sowas: wozu? Um dann mit den Reviewern zu streiten, bis das Paper so kritikabgesichert ist, dass keiner mehr den simplen Grundgedanken bemerkt? Das hilft nicht wirklich…

    @Dr. Webbaer: :-), eine schöne Wortschöpfung, “intellektuell obdachlos”.

  108. #108 MartinB
    1. September 2015

    @Benjamin
    Was der erste teil jetzt konkret mit der Behaupotung AB-Effekt durch Energieänderung zu tun hat, erschließt sich mir nicht.

    ” L ist nur relevant als linearer Faktor für den Gangunterschied der beiden Wellenzüge, er geht nur in einen der beiden Pfade ein, nicht in beide ”
    Aber in welchen? Und mit welchem Mehcanismus? Und da der Effekt ja von der Wewi mit dem B-Feld kommen soll, wie geht da die Weglänge ein? Ich habe nach wie vor keine Ahnung, wie du dir den Effekt über die B-Feld-Wewi vorstellst.

    “Um dann mit den Reviewern zu streiten, bis das Paper so kritikabgesichert ist, dass keiner mehr den simplen Grundgedanken bemerkt? Das hilft nicht wirklich… ”
    Ja, klar…

  109. #109 Benjamin Packisch
    2. September 2015

    @MartinB:
    Es interessiert nicht die absolute Länge, sondern der Unterschied der beiden Weglängen, aber ich wiederhole mich. Du kannst L z..B. als die kürzere der beiden Pfadlängen setzen – wohl wissend, dass die Pfade asymmetrisch bzgl des B-Feld-Bereichs verlaufen müssen (und die WP-Skizze hier irreführend ist). Asymmetrisch deshalb, weil das Integral über das Vektorpotential entlang der beiden Pfade jeweils unterschiedlich sein muss – so man die AB-Formel als Ausgangspunkt nimmt. Dann muss aber auch der B-Feldbereich bzgl. der beiden Pfade asymmetrisch sein. Und daraus folgt, dass das E-Feld der Elektronen unterschiedlich mit dem B-Feldbereich interagiert – und zwar weil das Elektronen-E-Feld sich nicht so perfekt abschirmen lässt wie das B-Feld, und sich so ein leichter Induktionseffekt im B-Feldbereich einstellt – der wiederum ein das Elektron bremsendes E-Feld gegeninduziert. Die Energie des Elektrons ist nun indirekt proportional zu seiner QM-Wellenlänge l. Aber auch hier wiederhole ich mich.

  110. #110 MartinB
    2. September 2015

    @benjamin
    “Asymmetrisch deshalb, weil das Integral über das Vektorpotential entlang der beiden Pfade jeweils unterschiedlich sein muss – so man die AB-Formel als Ausgangspunkt nimmt”
    Ist das so? Das wäre mir neu, die rechnung gibt das auch nicht her, soweit ich sehe:
    https://en.wikipedia.org/wiki/Aharonov%E2%80%93Bohm_effect
    Ich habe das jedenfalls seienrzeit im Studium für symmetrische Pfade hergeleitet, IIRC.

    “Du kannst L z..B. als die kürzere der beiden Pfadlängen setzen”
    Das scheint mir alles sehr willkürlich – warum soll auf dem “kürzeren Pfad” (wenn es denn einen gibt – eigentlich ist der Ab-Effekt doch topologisch und die Weg-Details spielen keine Rolle, s. berry-Phase) die Energie ne andere sein als auf dem langen.
    Und ist nun der kürzere Weg entscheidend oder der Unterschie der Weglängen?
    “Dann muss aber auch der B-Feldbereich bzgl. der beiden Pfade asymmetrisch sein.”
    Aber das B-Feld ist doch auf die Spule begrenzt – wieso spielt da der Pfad außen ne Rolle?

    Kannst du das alles bitte mal sauber vorrechnen und erklären, wie du dir den Mechanismus denkst? So ist das alles immer sehr unscharf, und ich habe nach wie vor keinen blassen Schimmer, wie du dir den Effekt durch Energieänderung erklären willst.

  111. #111 MartinB
    2. September 2015

    Nachtrag: Um es nochmal konkreter zu machen: Muss da nach deiner Idee nicht irgendwo so etwas wie ein Integral
    e int psi*(x) B(x) psi(x)
    oder so ähnlich eingehen, wenn da eine Energieänderung durchs B-Feld passieren soll, das mit der Ladungsdichte (e psi* psi) wechselwirkt?

  112. #112 Benjamin Packisch
    2. September 2015

    @MartinB:
    Langsam kommen wir der Sache wirklich näher, denn dank Deiner beharrlichen Einwände sehe ich jetzt etwas klarer. Vielleicht kann das folgende ein Konsens der offenen Frage sein:

    1) Für die Größe der Phasenverschiebung zwischen den zwei Pfaden a und b gilt diese Gleichung – ich denke das darf als Konsens vorausgesetzt sein.

    2) Wenn Du gestattest, hier eine anschauliche Deutung dieser Aussage, ausgehend vom ersten Maxwell-Paper:Das Vektorpotential A ist dort auf S. 481 als “electromagnetic momentum” (F;G;H) eingeführt (den Faraday “Electrotonic State” nannte).
    Wenn wir jetzt A ebenso fluiddynamisch deuten (und formal können wir das unzweifelhaft), dann kann man den AhBo-Versuchsaufbau sehr schön mechanisch deuten:
    Denn A ist genau analog zur Strömungsgeschwindigkeit, wenn B die Rotation des Strömungsfeldes ist. Gemäß dem Biot-Savart-Gesetz und den Arbeiten von Helmholtz ist dann das eingeschlossene B-Feld exakt analog zu einem Wirbelfaden mit Rotation, und das umgebende v-Feld ist eine Rotationsbewegung, die mit 1/r abfällt und damit rotationsfrei ist. Das ist die anschaulich gemachte Situation der WP-Skizze mit den beiden symmetrischen Pfaden. Der eine Pfad a läuft mit A, der andere b dagegen.
    Theoretisch bräuchte man also einen unendlich langen, aber geschlossenen B-Faden, um den AB-Effekt zu demonstrieren und auszuschließen, dass vom einen Pfad im Schnitt ein anderer Energiebetrag auf den B-Faden übergeht als vom anderen.

    3) Zu Deiner Gleichung aus #111: Der Term ist natürlich immer Null, wenn man annimmt, dass Psi und B-Faden perfekt abgeschirmt sind. Die Frage ist aber, mit welchem Argument wir einerseits
    I)einen (asymmetrischen, sehr langwelligen) Photonenaustausch zwischen den e-Pfaden und dem B-Faden
    und andererseits
    II) die Wirkung unquantisierter Induktion beim AB-Effekt
    ausschließen können und ob
    III) die E-Feld-Wirkung des Elektron-Psi genauso perfekt abgeschirmt ist wie der B-Faden andersherum (Stichwort Tunneln)?
    Weißt Du vielleicht wie man ersteres sauber aufschreiben würde? Also wie man die Emission eines Photons aus dem Elektron über das E-Feld des Elektrons in zweiter Quantisierung formuliert? Vermutlich ist das allgemein irgendwo publiziert…
    Und würdest Du zustimmen, dass diese drei Punkte die zentrale Frage unseres Disputes ist?
    Für jeden Hinweis wäre ich dankbar.

  113. #113 MartinB
    2. September 2015

    @Benjamin
    Ich stimme dir zu, dass der Knackpunkt natürluich darin steckt, dass der eine Pfad mit A läuft, der andere dagegen.
    “Theoretisch bräuchte man also einen unendlich langen, aber geschlossenen B-Faden, um den AB-Effekt zu demonstrieren und auszuschließen, dass vom einen Pfad im Schnitt ein anderer Energiebetrag auf den B-Faden übergeht als vom anderen.”
    Sehe ich nicht so strikt: Wenn ich zeigen kann, dass der AB-Effekt unabhängig davon ist, wie ich die Elektronen um die Spule herumführe (also dicht dran oder weit weg), dann sehe ich nicht, wie man das auf eine Wewi mit B (und nicht mit A) zurückführen können soll. (Quantitativ, nicht qualitativ.)
    Deswegen ist auch ” Der Term ist natürlich immer Null, wenn man annimmt, dass Psi und B-Faden perfekt abgeschirmt sind.” nicht der Punkt – selbst wenn der Term nicht Null ist, ist er doch sicher in den verschiedenen AB-Experimenten ganz unterschiedlich, es dürfte also schwer sein, dasselbe Ergebnis zu bekommen.

    ” Die Frage ist aber, mit welchem Argument wir einerseits
    …ausschließen können ”
    Nein, das ist letztlich nciht die Frage – solange es keine Rechnung gibt, die zeigt, dass man den AB-Effekt auf diese alterntaive (oder eine andere) Weise quantitativ korrekt abbilden kann, müssenw ir nicht nachweisen, dass das unmöglich ist – das ist letztlich Beweislastumkehr.

    ” Also wie man die Emission eines Photons aus dem Elektron über das E-Feld des Elektrons in zweiter Quantisierung formuliert? ”
    Ich weiß nicht, was du da schreiben willst – nen Feynman-Diagramm, bei dem das Elektron ein Photon aussendet, das dann bei der Spule ankommt und da – was tut? Mit den Elektronen im Draht der Spule wechselwirkt?

    Ich sag’s nochmal: ich habe – auch nach diesem Kommentar – wirklich keine Idee, wie du dir die Wewi zwischen Elektron und Spule vorstellst und wer da wie Energie übertragen soll – das Elektron auf die Spule? Die Spule (via Magnetfeld oder Vektorpotential?) auf das Elektron? Und ich sehe auch nicht, wie da die zweite Quantisierung viel hilft – im Grenzfall niedriger Energien etc. kommt für die Wewi doch wieder die SGl heraus, oder wo siehst du da Möglichkeiten für einen zusätzlichen Effekt?

  114. #114 Benjamin Packisch
    3. September 2015

    @MartinB.
    Warum ist denn für Dich konkret das Ampère’sche Gesetz der Maxwell-Gleichung nicht relevant für die Wechselwirkung zwischen dem elektrischen Feld des Elektrons und und dem B-Feld des Fadens? Ich habe das schon gefühlt zehnmal geschrieben, aber bisher nur gehört : “sehe ich nicht”.
    Quantisiert würde das Elektron auf dem einen Pfad ein Photon emittieren, weil sein E-Feld mit dem B-Faden überlappt, und dadurch würde es Energie verlieren.
    Also, wenn Du dazu einen konkreten Satz schreiben könntest, wäre das aufschlussreich.

    Noch eine Frage zu folgendem:
    “… solange es keine Rechnung gibt, die zeigt, dass man den AB-Effekt auf diese alterntaive …Weise quantitativ korrekt abbilden kann, müssen wir nicht nachweisen, dass das unmöglich ist …”
    Wen meinst Du mit “wir”? Die akademische Physik als Ganzes? Oder Dich als Vertreter all ihrer “Ideale”?
    MIr will es ziemlich maßlos erscheinen, die Möglichkeit von Alternativen geradezu juristisch (“Beweislastumkehr”) von sich zu weisen oder sie pauschal als irrelevant abzutun…
    Außerdem bedeutet die Existenz qualitativer Alternativen (bei quantitativer Äquivalenz) ja nicht, dass “alle Physiker ihr Leben ändern” müssten
    Kannst Du das vielleicht kommentieren?

  115. #115 MartinB
    4. September 2015

    @Benjamin
    ” die Wechselwirkung zwischen dem elektrischen Feld des Elektrons und und dem B-Feld des Fadens”
    Ich habe das aber schon 10mal geschrieben, eben sogar mit Formeln zu erklären versucht. Wenn diese Abhängigkeit da wäre, dann müsste der Effekt doch vom Weg abhängen – je weiter ich das Elektron um die Spule herumführe, desto schwächer müsste der Effekt ja werden, weil das E-Feld des Elektrons dann am Ort der Spule immer schwächer wird. Das geben die Experimente, soweit ich sehe, nicht her. Wie genau soll die Formel für den Effekt aussehen, bei der am Ende wegunabhängig die Formel für den AB-Effekt herauskommt?

    “MIr will es ziemlich maßlos erscheinen, die Möglichkeit von Alternativen geradezu juristisch (“Beweislastumkehr”) von sich zu weisen”
    Du verstehst (trotz meines Textes oben) anscheinend nicht, wie Physik funktioniert: Du hast – wenn wir ernsthaft physikalisch argumentieren – noch keine Argumente vorgetragen, die würden sich nämlich in einer Rechnung niederschlagen, in der du zeigst, dass deine idee (“Ich glaube, man kann den AB-Efffekt auch direkt durch Wewi zwischen E-Feld und B-Feld erklären”) auch tatsächlich funktioniert. *Wenn* du das kannst, *dann* wäre es an den Vertretern der herkömmlichen Interpretation des AB-Effekt,s zu zeigen, dass deine Rechnung oder Interpretation fehlerhaft ist oder anderweitig zu falschen Folgerungen führen würde. Hast du aber nicht getan, du hast letztlich also keine Evidenz geliefert.

    Ich habe übrigens gerade das hier gefunden:
    https://arxiv.org/pdf/1009.1613
    https://link.springer.com/article/10.1007%2FBF02730630
    aber mangels Zeit nicht gelesen. Vielleicht hilft es ja weiter.

    “…bei quantitativer Äquivalenz…”
    Genau das hast du ja nicht gezeigt. Beispiele für unterschiedliche Interpretationen einer Theorie (QM) habe ich ja oben kurz angesprochen, so ein Fall liegt hier aber nicht vor, ebensowenig wie bei deinem anderen Beispiel mit der SRT.

  116. #116 MartinB
    4. September 2015

    PS:
    Noch ein Nachtrag: Ich habe nichts gegen Erlärungen, die von den herkömmlichen abweichen. In den meisten Büchern steht ja z.B., dass der Casimir-Effekt ein Nachweis der Vakuumfluktuationen ist, in meiner Vakuum-Serie habe ich aber gezeigt, warum dieses Argument nicht 100% wasserdicht ist.

    Genauso habe ich auch kein Problem, wenn jemand tatsächlich zeigen kann, dass man den AB-Effekt anders erklären kann (wobei er trotzdem eine spektakuläre Vorhersage der SGL bleibt, deswegen hatte ich ihn ja hier angeführt, als ein schönes gegenbeispiel gegen deine ursprünglichen Behauptung, die SGl könne ja nur das H-Atom ohne Zusatzannahmen korrekt berechnen (was ich nach wie vor für ausgemachten Blödsinn halte)). Die müssen halt nur quantitativ funktionieren und konsistent sein.

  117. #117 Benjamin Packisch
    4. September 2015

    @MartinB:
    ” Ich habe das aber schon 10mal geschrieben … Wenn diese Abhängigkeit da wäre, dann müsste der Effekt doch vom Weg abhängen”.
    Konventionell hängt die Phasenverschiebung vom Weg-Integral über A ab, also von int A dx. Das ist es, was diese Gleichung aussagt, und es zeigt sich aber offenbar erst durch eine anschauliche Deutung von A unmißverständlich. Das heißt, dass Pfade näher am B-Faden – egal in welcher Erklärung – einem stärkeren AB-Effekt unterworfen sind, als solche weiter weg. Überhaupt messbar wird der Phasenunterschied, weil zwei Pfade P1 und P2 unterschiedliche Phasenversätze aufweisen.
    Obwohl A lokal rotationsfrei ist, ist die Integration entlang der Pfade eben doch wegabhängig, weil sie ein Gebiet mit Rotation umschließen.

    Das arxiv-Papier werde ich gerne lesen, auf da andere habe ich keinen Zugriff.

    P.S.: Zur Konsistenzforderung bei alternativen Erklärungen: Wir sind uns einig, dass das Urteil ” xy ist konsistent” von der Sprecher-Kompetenz über xy abhängt. Für ein akzeptables physikalisches Konsistenzurteil forderst Du im Artikel Mathematik. Fehlt sie, folgerst Du, dass die Theorie Unsinn oder unausgegoren sei.
    Ich bin aber nach wie vor der Ansicht, dass diese Forderung allein irreführend ist.
    Liegt z.B. eine alternative Theorie mit Mathematik vor, dann muss der Abweichler einen schier unglaublichen Wust an konventioneller Gewohnheit und Abwehr überwinden, und zum Beispiel immer neue Nachweise erbringen, denen keine etablierte Theorie im Detail je gerecht werden könnte oder die auch nur gefordert würden. Diese Diskrepanz zwischen Forderungen an Alternativen und Vertrauen in Etabliertes halte ich für bedenklich in der Wissenschaft – und vor allen Dingen für unnötig.
    Was ist der wissenschaftliche Sinn von Denk-Schemata? Wodurch ist die Verteilung von Vertrauen und Mißtrauen in Deutungen formaler Konzepte begründet?

    Warum zum Beispiel wird üblicherweise gelehrt, dass A eine mathematische Hilfskonstruktion ohne Anschauung sei, obwohl schon Faraday und Maxwell A als anschauliche Größe eingeführt hatten? War Dir z.B. dieser Punkt bekannt?

  118. #118 MartinB
    4. September 2015

    @Benjamin
    ” Das heißt, dass Pfade näher am B-Faden – egal in welcher Erklärung – einem stärkeren AB-Effekt unterworfen sind, als solche weiter weg. ”
    Aber am Ende hebt sich das ja alles brav auf und nur der magnetische Fluß bleibt übrig, weil zum einen der Weg länger qwird und das Vektorpotential schwächer (ähnlich wie beim Flussgesetz in der Elektrostatik).

    Wenn aber das Feld des Elektrons mit dem B-Feld wechselwirkt, dann gilt das nicht mehr.

    “ür ein akzeptables physikalisches Konsistenzurteil forderst Du im Artikel Mathematik. Fehlt sie, folgerst Du, dass die Theorie Unsinn oder unausgegoren sei.”
    Naja, wer Messergebnisse mit Theorien erklären will, der muss schon Zahlen produzieren.

    ” diese Forderung allein ”
    Von *allein* kann ja nicht die rede sein.

    ” dann muss der Abweichler einen schier unglaublichen Wust an konventioneller Gewohnheit und Abwehr überwinden”
    Nein. Das sieht man ja zum beispiel am durchaus leicht zu findenden paper von Jaffe über den Casimir-Effekt.

    “Diese Diskrepanz zwischen Forderungen an Alternativen und Vertrauen in Etabliertes halte ich für bedenklich in der Wissenschaft ”
    Letztlich ist das aber genau das “Schultern-von-Riesen-Prinzip” von Newton.

    “Warum zum Beispiel wird üblicherweise gelehrt, dass A eine mathematische Hilfskonstruktion ohne Anschauung sei,”
    Ich denke, das liegt (gerade in der klassischen Physik) an der Eichfreiheit und daran, dass es klassisch soweit ich weiß keine Effekte gibt, wo Ladungen mit A direkt wechselwirken. Meiner Ansicht nach sollte man allerdings auch in Kursen zur klassischen E-Dynamik mal den AB-Effekt erwähnen und erklären, dass das vektorpotential in der QM (und erst recht in der QFT) eine zentrale Rolle einnimmt.

    Es gibt seeeehr viele Punkte, wo ich mit der üblichen Erklärung in der Physik nicht einverstanden bin – ein anderes Beispiel ist mein Artikel von neulich zum Heisenberg- und Schrödinger-Bild; von vielen Dingen in der QFT ganz zu schweigen (da habe ich ja extra von den “Verwirrungen der QFT” gesprochen, und dass man in der kanonischen Formulierung nie auch nur ansatzweise erklärt, was ein Zustand eigentlich ist, finde ich auch wenig hilfreich).

    “War Dir z.B. dieser Punkt bekannt?”
    Nein, aber ich bin auch nicht in allen Aspekten der Wissenschaftsgeschichte besodners firm, und finde auch nicht immer relevant, was Leute vor 150 Jahren gedacht haben – Maxwell musste auch seine Gleichungen in Komponentenschreibweise hinschreiben, weil die Vektorschreibweise damals noch nicht etabliert war.

    Ich kann es nur noch mal sagen: Ich habe kein Problem damit, wenn jemand – gut begründet – bekannte Ideen neu deutet oder Alternativen formuliert. Die müssen aber schon konkret sein (und das heißt eben auch: Nachweislich mit Messergebnissen übereinstimmen.) Für den AB-Effekt sehe ich das nicht (obwohl es da wohl Ideen gibt, siehe auch das paper.)

    Ich habe aber ein Problem, wenn du Dinge sagst wie
    “Die SGL klappt nur für’s H-Atom.” oder
    “Ich kenne einen E-Techniker, der findet, dass man den AB-Effekt auch über ne Kraft erklären können müsste, und glaube auch, dass das irgendwie geht.”
    Das ist in meinen Augen keine fundierte Kritik – warum, habe ich ausführlich begründet.

  119. #119 Bjoern
    4. September 2015

    @Benjamin:

    Liegt z.B. eine alternative Theorie mit Mathematik vor, dann muss der Abweichler … immer neue Nachweise erbringen, denen keine etablierte Theorie im Detail je gerecht werden könnte oder die auch nur gefordert würden.

    Für diese Behauptung hätte ich gerne mal Belege.

    Ich kenne es eigentlich nur so, dass, wenn jemand mit einer alternativen Theorie ankommt, diese erst mal _dasselbe_ erklären können muss wie die etablierte Theorie. Wenn sie das kann, dann wird sie als alternative Theorie akzeptiert – und erst dann kommt die Frage, was denn an dieser neuen Theorie _besser_ ist als an der alten, sprich: was sie _zusätzlich_ erklären kann.

  120. #120 Benjamin Packisch
    7. September 2015

    @MartinB:
    ” ” Das heißt, dass Pfade näher am B-Faden – egal in welcher Erklärung – einem stärkeren AB-Effekt unterworfen sind, als solche weiter weg. ”
    Aber am Ende hebt sich das ja alles brav auf und nur der magnetische Fluß bleibt übrig….”
    Nein, int A dx hebt sich eben nicht auf, weil man ja gerade nicht im Kreis integriert, sondern zwei Teilpfade vergleicht. Die Gleichung ist gerade kein Integral über B sondern über A, und sie ist auch kein Kreisintegral. Und was ist nun mit dem Elektron des E-Feldes? Wie weit reicht es?

    Ich sehe den EInwand des Elektrotechnikers noch immer nicht entkräftet – auch wenn ich es sehr interessant fände, wenn tatsächlich unmissverständlich der Maxwell’sche Äther-Impuls im AB-Effekt wirken würden ;-).

    …Und Du wirst mir womöglich zustimmen, dass die Diskussion von Alternativen nicht erleichtert wird, wenn offensichtlich schon die konventionelle Erklärung nicht eindeutig ist.

    Zur SGL: Das Problem, wann Elektronen in Feldern strahlen, und wann nicht, wird durch die SGL m.E. nicht erschöpfend erklärt, sondern es geht als Voraussetzung in die Gleichung ein, um z.B. das H-Atom zu quantifizieren. Wenn Dir dieser Einwand das nicht fundiert genug beschrieben ist, ok, aber ein Problem bleibt es doch.

    @Bjoern: Schau Dir einfach mal spielerisch den Subtext der Diskussion oben an.
    Wenn eine neue Theorie wirklich “dasselbe” erklären würde wie eine alte, dann wäre sie nicht wirklich neu. Diese Forderung würde bedeuten, dass man etwas Neues nur formulieren dürfte, wenn man der gesamten vorhergegangenen Tradition und ihren Deutungen und Messinterpretationen zustimmen würde.
    Neue Theorien bedingen aber mindestens in den Deutungsgewohnheiten Brüche.
    Und die Form der Deutung beeinflusst die Art die Messung, wie man schon bei Kant lernen konnte.
    Oder wie Planck es sinngemäß beschrieben hat: Die Vertreter alter Lehrmeinungen werden nicht überzeugt, sondern sie sterben aus.

  121. #121 Bjoern
    7. September 2015

    @Benjamin:

    Nein, int A dx hebt sich eben nicht auf, weil man ja gerade nicht im Kreis integriert, sondern zwei Teilpfade vergleicht. Die Gleichung ist gerade kein Integral über B sondern über A, und sie ist auch kein Kreisintegral.

    Beobachtbar ist nur die Phasendifferenz der beiden Pfade. Diese Phasendifferenz ergibt sich als Differenz der Integrale entlang der beiden Teilpfade. Und das ergibt insgesamt wiederum ein Integral entlang eines geschlossenen Wegs. Und ein geschlossenes Wegintegral über A kann man mit dem Satz von Stokes umschreiben in ein Flächenintegral über B. Insgesamt stellt man fest, dass _alle_ beobachtbaren Effekte letztlich nur vom magnetischen Fluss abhängen.

    Oder, kurz gesagt: Martin hat recht.

    Schau Dir einfach mal spielerisch den Subtext der Diskussion oben an.

    Ja. Und? Ich sehe da keinen Beleg für deine Behauptung. Wie wäre es, wenn du ein konkretes Beispiel bringen würdest?

    Wenn eine neue Theorie wirklich “dasselbe” erklären würde wie eine alte, dann wäre sie nicht wirklich neu.

    Warum??? Wenn sie dasselbe erklärt wie das alte, aber auf andere Weise, dann ist sie doch neu!

    Diese Forderung würde bedeuten, dass man etwas Neues nur formulieren dürfte, wenn man der gesamten vorhergegangenen Tradition und ihren Deutungen und Messinterpretationen zustimmen würde.

    Nein, das folgt nicht im mindesten. Man muss nur die vorhergegangenen Mess_ergebnissen_ berücksichtigen.

    Und die Form der Deutung beeinflusst die Art die Messung, wie man schon bei Kant lernen konnte.

    Die Form der Deutung ändert aber exakt überhaupt nichts an den Mess_ergebnissen_.

    Oder wie Planck es sinngemäß beschrieben hat: Die Vertreter alter Lehrmeinungen werden nicht überzeugt, sondern sie sterben aus.

    Weder dass der Spruch immer wieder zitiert wird, noch dass er von einem wissenschaftlichen Genie stammt, macht ihn richtig. Es gibt mehr als genug Gegenbeispiele zu dieser Behauptung.

  122. #122 MartinB
    7. September 2015

    @Benjamin
    Soweit ich sehe, hat Bjoern zum AB-Effekt alles notwendige gesagt.
    Wo da jetzt noch der Äther ins Spiel kommen soll, erschließt sich mir nicht.

    “wann nicht, wird durch die SGL m.E. nicht erschöpfend erklärt, sondern es geht als Voraussetzung in die Gleichung ein, um z.B. das H-Atom zu quantifizieren. ”
    Nein, es geht wie oben erläutert nicht ein. Ein Elektron im Grundzustand kann nicht strahlen, weil es keine Energie verlieren kann – deswegen heißt der Grundzustand Grundzustand. Und dass es im H-Atom einen Grundzustand gibt, ist eine direkte Folge der SGL. Egal wie oft du behauptest, da sei ein Problem – es ist keins.

    “dass die Diskussion von Alternativen nicht erleichtert wird, wenn offensichtlich schon die konventionelle Erklärung nicht eindeutig ist.”
    Huh? Weil es bei der konventionellen Erklärung schon Alternativen gibt (nehmen wir mal an, das wäre so), wird die Diskussion weitere Alternativen erschwert? Warum soll das so sein? Ich schließe mich Bjoerns Aufforderung an: Kannst du mal ein konkretes Beispiel bringen für das, was du meinst? Gerade die Qm mit ihrer Vielzahl von Deutungen zeigt doch das Gegenteil: neue Alternativen werden durchaus diskutiert.

    “Wenn eine neue Theorie wirklich “dasselbe” erklären würde wie eine alte, dann wäre sie nicht wirklich neu.”
    Kannst du dich bitte auch mal einigen, worüber wir nun diskutieren – über alternative, in den Ergebnissen äquivalente Interpretationen oder Darstellungen, oder über unterschiedliche Theorien mit unterschiedlicher Erklärungskraft?

    “Und die Form der Deutung beeinflusst die Art die Messung, wie man schon bei Kant lernen konnte.”
    Wo Kant auch nur ansatzweise so etwas gesagt hat, würde mich sehr interessieren – synthetische Urteile a priori sind ja keine “Deutung”.

  123. #123 Benjamin Packisch
    8. September 2015

    @MartinB & @Bjoern:
    Okay, ihr habe insofern recht, als dass die Phasendifferenz am Messschirm nur vom umschlossenen rotA abhängt. Die Phase selbst ist aber jeweils durchaus vom Pfad abhängig – und es wäre möglich, sie einmal mit und einmal ohne B-Faden zu vermessen, und daraus die
    absoluten Phasendifferenzen beider Pfade zu bestimmen. Die Gleichung macht dazu ja eine explizite Aussage.

    Das ändert alles nichts an der Frage, was mit dem E-Feld des Elektrons nun ist. HIer steht nach wie vor unversöhnt Aussage gegen Aussage – wobei ich mich mit einem guten Argument gern überzeugen lassen. Ich fände eine einheitliche Physik ohne Deutungsdivergenzen auch schöner – die Frage ist, wie hoch der Preis dafür ist.

    Da ihr beide noch etwas Ähnliches gefragt habt, hier meine zusammengefasste Antwort:
    “Und die Form der Deutung beeinflusst die Art die Messung, wie man schon bei Kant lernen konnte.”
    Wo Kant auch nur ansatzweise so etwas gesagt hat, würde mich sehr interessieren – synthetische Urteile a priori sind ja keine “Deutung”.

    Wo Kant das sagt, habe ich schon mehrfach zitiert, aber offenbar unbemerkt, also nochmal: HIER!
    Die Textpassage ist nicht ganz leicht zu verstehen, aber ihre Tragweite ist enorm und leider bis heute nur sehr selten wirklich konsequent begriffen.

    Martin sagt, synthetische Urteile a priori (“sUap”) seien keine Deutungen. Ich bin anderer Meinung: Ein sUap ist geradezu der Prototyp einer Deutung, d.h. einer Zuschreibung, die sich primär NICHT aus der Erfahrung (d.h. Messung) begründet. Deutungen beziehen sich auf die Struktur unseres Denkens und Urteilens, nicht auf die Physis.
    Ein Beispiel dafür ist die die Aufteilung von Messwerten in in ideale Messgrößen und Störgrößen (auf Grundlage von Hypothesen, die primär nie aus der Erfahrung stammen, sondern aus vorläufigen Behauptungen)

    @Bjoern:
    Warum??? Wenn sie dasselbe erklärt wie das alte, aber auf andere Weise, dann ist sie doch neu!
    Ja, aber Du kannst selbst z.B. bei MartinB lesen, dass Alternativen in konventionellen Erklärung nicht beliebt sind. Die einheitliche Sichtweise ist ein starkes Ideal der Physik. Das heißt, alternative Sichtweisen treffen auf einen Widerstand, der m.E. am besten durch dieses Zitat charakterisiert ist:
    “Der Wille liegt tiefer als die Erkenntnis, und Gewohnheit macht den Willen stark.” (Ernst Barthel)

    Ich vermute, Du hättest es lieber etwas konkreter: die zwei Beispiele hier (Sagnac & AB) sind es, und an Ihnen kannst Du außerdem den genannten Widerstand studieren.

  124. #124 MartinB
    8. September 2015

    @Benjamin
    ” wobei ich mich mit einem guten Argument gern überzeugen lassen.”
    Das ist tatsächlich experimentell widerlegt – das gute Argument habe ich oben mehrfach gebracht (wenn das E-Feld mit dem B-Feld wechselwirkt, dann würde sich diese Wechselwirkung ändern, wenn ich den Weg des Elektrons um die Spule ändere, tatsächlich bleibt der AB_Effekt aber unverändern.)

    “Ein sUap ist geradezu der Prototyp einer Deutung”
    Sehe ich nicht so, weil ein sUap laut Kant eine *Denknotwendigkeit* ist – während zumindest für mich das Wort “Deutung” etwas anderes impliziert.

    Die anderen Fragen (auch die von Björn) hast du nicht wirklich beantwortet, und warum weder Sgnac noch AB als Beispiele taugen, haben wir ja mehrfach erläutert.

  125. #125 Bjoern
    8. September 2015

    @Benjamin:

    Die Phase selbst ist aber jeweils durchaus vom Pfad abhängig – und es wäre möglich, sie einmal mit und einmal ohne B-Faden zu vermessen, …

    Nein, das ist nicht möglich – die Phase der Wellenfunktion ist eine prinzipiell unbeobachtbare Größe. Beobachtbar sind nur Phasen_differenzen_.

    Das ist doch genau der wesentliche Punkt der U(1)-Eichinvarianz!

    Die Gleichung macht dazu ja eine explizite Aussage.

    Auch das stimmt nicht. Die Schrödinger-Gleichung legt die Phase der Wellenfunktion _nicht_ fest.

    Wieder: U(1)-Eichinvarianz!

    Ja, aber Du kannst selbst z.B. bei MartinB lesen, dass Alternativen in konventionellen Erklärung nicht beliebt sind.

    Martin hat nichts dergleichen gesagt. Sowohl hier als auch in diversen Artikeln hat er eigentlich das genaue Gegenteil gesagt: Dass es für vieles in der Physik unterschiedliche, gleichwertige Deutungen gibt, und dass das auch sinnvoll ist.

    Ich vermute, Du hättest es lieber etwas konkreter: die zwei Beispiele hier (Sagnac & AB) sind es, und an Ihnen kannst Du außerdem den genannten Widerstand studieren.

    Das Beispiel mit Sagnac muss ich mir noch anschauen – aber zumindest bei AB ist der Widerstand absolut gerechtfertigt: die “alternative Deutung” ist schlicht falsch, wie von Martin eindeutig dargelegt wurde.

  126. #126 Benjamin Packisch
    8. September 2015

    @MartinB:
    Du sagst: die Wegabhängigkeit des AB-Effekts sei experimentelll widerlegt: Darf ich um eine Referenz bitten? MIr ist noch keine Messung bekannt, die das explizit untersucht hat. Wie gesagt, man könnte den Einfluss des Magnetringes auf die Phase schon bestimmen, in dem man den Schirm einmal mit magnetisiertem Ring und einmal mit unmagnetisiertem im Strahlengang belichtet. Oder auch mit unterschiedlich starken Magnetisierungen. Wenn das schon irgendwo vermessen ist, wäre ich für einen Hinweis dankbar.

    Hast Du derweil die Kant-Passage gelesen? Ich zitiere nochmal vom Ende, weil ich den Eindruck habe, dass Du nicht wirklich zur Kenntnis nehmen willst, was Kant da geäußert hat:
    … weil Erfahrung selbst eine Erkenntnisart ist, die Verstand erfordert, dessen Regel ich … a priori voraussetzen muß, welche in Begriffen a priori ausgedrückt wird, nach denen sich also alle Gegenstände der Erfahrung notwendig richten … .
    Was Gegenstände betrifft, sofern sie bloß durch Vernunft … gedacht, die aber … gar nicht in der Erfahrung gegeben werden können, so werden die Versuche sie zu denken …, hernach einen herrlichen Probierstein desjenigen abgeben, was wir als die veränderte Methode der Denkungsart annehmen, daß wir nämlich von den Dingen nur das a priori erkennen, was wir selbst in sie legen.

    Das heißt unmissverständich: Die a priori-Anteile der “Dinge” unterliegen unserem freien Willen, und zwar als notwendige, aber aktive Bestandteile des Denkens. Deutungen sind klipp und klar notwendige und aktive a priori Bestandteile jeglicher Erkenntnis. Messungen können diese a priori Bestandteile dann nur noch präzisieren, aber nicht mehr grundsätzlich ändern. Das erkennt man beispielsweise in der Art Messungen, die üblicherweise gemacht oder eben ausgelassen werden.

    @Bjoern:
    Bitte lies doch erstmal genau den vorangegangen Dialog, damit Du weißt, welche Gleichung ich meinte.
    Dass es in der Physik ernsthaft praktizierte Deutungsfreiheit gäbe, ist mir durchaus noch nicht aufgefallen. Wenn es sie gibt, dann offenbar nur, wenn sie “längst von Experten anerkannt” ist, z.B. steriles und irrelevantes Kuriosum ;-).
    Bei etwas “absolut gerechtfertigtem”, wie Du es nennst, muss ich daher Dogmenverdacht zumindest erwägen…

  127. #127 MartinB
    8. September 2015

    @Benjamin
    “Du sagst: die Wegabhängigkeit des AB-Effekts sei experimentelll widerlegt”
    Nein, ich habe gesagt, dass experimentell widerlegt ist, dass du bei guten gegenargumenten deine Meinung änderst.
    ” Darf ich um eine Referenz bitten?”
    Da keine der Arbeiten explizit den Weg in die Berechnung einbeziehen muss, dient eigentlich jede als referenz.

    Was die Kant-Passage angeht – wo steht da etwas vom freien Willen? Ich verstehe die kant-Passage so, dass er die allgemein bekannten geänderten Denkweisen, die zu geänderten Anschauungen und Interpretationen von Ergebnissen führen, als beispiel/Analogie hernimmt um zu argumentieren, dass dasselbe auch für a-priori-Vorstellungen gilt. Dass Kant jemals geäußert hätte, a-priorische Urteile und Vorstellungen unterlägen dem freien Willen, wäre mir neu, es widerspricht auch allem, was ich von Kant gelesen habe (wenn da freier Wille im Spiel wäre, könntenw ir uns bei den Antinomien der reinen Vernunft a einfach für eine Lösung entscheiden und wären dann zufrieden, genau das ist ja nicht der Fall). “Was wir selbst in sie legen” kann deshalb in meinen Augen nicht als “bewusst” oder “mit freiem Willen” interpretiert werden,

    “Wenn es sie gibt, dann offenbar nur, wenn sie “längst von Experten anerkannt” ist, z.B. steriles und irrelevantes Kuriosum”
    O.k., langsam wird’s witzig (oder trollig). “Ich kenne keine Fälle, aber wenn es welche gibt, dann zählen sie sowieso nicht, weil ich sie sonst kennen würde.” Ja, das ist echt überzeugend argumentiert. (Insbesondere angesichts der Tatsache, dass ich mit den Interpretationen der Qm ja genau so etwas längst angesprochen habe.)

  128. #128 Benjamin Packisch
    8. September 2015

    @MartinB.
    “Nein, ich habe gesagt, dass experimentell widerlegt ist, dass du bei guten gegenargumenten deine Meinung änderst.”
    Das gleiche kann ich von Dir auch behaupten. Hier stößt Meinung auf Meinung, und Deine Meinung behauptet, die gute zu sein. Na wenigstens haben wir es bis zur Ebene der Moral geschafft.
    Der Artikel, den Du mir geschickt hast, ist interessant, aber er hilft nicht weiter, weil er am entscheidenden Punkt ausweicht: er blendet die Möglichkeit (z.B. evaneszenter) E-Feldstreuung in den B-Faden aus.

    Die kurze Google-Suche offenbart Dir keinen “freien Willen” in der Kant-Passage, als folgerst Du, es stünde nichts darüber drin. Das könnte ich nun wieder lustig finden, wenn es nicht so tragisch wäre.

    Die Anwendung der a priori Urteile unterliegt dabei dem freien Willen, also die Frage, bis zu welchem Preis ich eine Hypothese verteidige oder nicht.
    Es ist schlicht ein Irrtum oder sogar eine Lüge, dass es alternativlose Argumente für eine x-beliebige Hypothese gäbe. Ohne faktisch unbegründeten guten (oder schlechten) Willen nützt die schönste Theorie nichts, weil man ihr immer widersprechen kann. Ob wir dieses Prinzip hier in “guter” oder “schlechter” Absicht benutzt haben, sei dahingestellt, aber demonstriert haben wir es hinlänglich.

    Wenn Du meinen Hinweis zum Blick auf die Voraussetzungen physikalischer Aussagen verweigerst, dann tu es bitte mit dem einzigen soliden Argument, gegen das ich wirklich nichts einwenden kann: Du willst nicht.

  129. #129 MartinB
    8. September 2015

    @Benjamin
    “er blendet die Möglichkeit (z.B. evaneszenter) E-Feldstreuung in den B-Faden aus. ”
    Ja, und wenn er trotzdem zum richtigen Ergebnis kommt, dann ist
    1. dieser Weg damit doch als mögliche Erklärung etabliert und
    2. sehr fraglich wie man (angesichts der Tatsache, dass eben nirgends ein Integral der Art int psi* psi B in die rechnung eingeht) den Effekt auf eine WeWi mit dem E-Feld zurückführen kann. Den Beweis, dass das geht, bist du bisher ja schuldig geblieben.

    “Die Anwendung der a priori Urteile unterliegt dabei dem freien Willen”
    Dass Kant das jemals gesagt hätte, wäre mir neu (und ich habe zwar nicht dieganze Kritik der reinen Vernunft gelesen, aber auch keinen vernachlässignaren Anteil, einige teile auch mehrfach). Synthetische Urteile a priori sind doch Urteile, zu denen wir ohne Erfahrung *zwangsläufig* kommen – Beispiele sind ja etwa raum udn Zeit, die laut kant eben Denknotwendigkeiten sind und nicht anders gedacht werden können.
    “Das könnte ich nun wieder lustig finden, wenn es nicht so tragisch wäre. ”
    Das könnte ich nun wieder lustig finden, wenn es nicht so arrogant wäre (und zeigen würde, dass du meinen Text nicht wirklich gelesen hast)

    “Es ist schlicht ein Irrtum oder sogar eine Lüge, dass es alternativlose Argumente für eine x-beliebige Hypothese gäbe”
    Habe ich das irgendwo behauptet? Aber aus der Tatsache, dass es Alternmativen geben kann, folgt weder, dass es sie tatsächlich gibt, noch, dass irgendeine x-beliebige durch nichts gestütze Aussage eine Alternative ist. Deswegen frage ich ja konkret nach.

    “tu es bitte mit dem einzigen soliden Argument, gegen das ich wirklich nichts einwenden kann:”
    O.k., du sagst also deutlich, dass du nicht bereit bist, dich von Argumenten überzeugen zu lassen – da du ja etwas anderes als ein “Ich will nicht” von mir schon von vornherein nicht als Argument akzeptierst. Immerhin gibst du es zu.

    Aber gut. Dann lassen wir’s.

  130. #130 Bjoern
    8. September 2015

    Dass es in der Physik ernsthaft praktizierte Deutungsfreiheit gäbe, ist mir durchaus noch nicht aufgefallen.

    Dann hast du echt keine Ahnung, von was du redest. Bestes Beispiel (was auch schon mehrfach erwähnt wurde!): verschiedene Deutungen der Quantenmechanik.

    Bitte lies doch erstmal genau den vorangegangen Dialog, damit Du weißt, welche Gleichung ich meinte.

    Ich muss nicht den ganzen Dialog lesen, um zu sehen, dass deine Behauptungen zum AB-Effekt offensichtlich falsch sind – weil sie nämlich jeder Menge seit ca. 90 Jahren exzellent durch Messungen etablierter Physik widersprechen.

    Wie gesagt, man könnte den Einfluss des Magnetringes auf die Phase schon bestimmen, in dem man den Schirm einmal mit magnetisiertem Ring und einmal mit unmagnetisiertem im Strahlengang belichtet.

    Nein, mit so einem Versuch kann man nur den Einfluss des Magnetringes auf die Phasen_DIFFERENZ_ bestimmen. (und meines Wissens wurden die Versuche zum AB-Effekt sogar genau so durchgeführt!)

  131. #131 Bjoern
    8. September 2015

    @Benjamin:

    Dass es in der Physik ernsthaft praktizierte Deutungsfreiheit gäbe, ist mir durchaus noch nicht aufgefallen.

    Gleich noch ein zweites Beispiel: Die klassische Bewegung von Teilchen auf Bahnen kann man einerseits mittels Kräften beschreiben – oder andererseits durch Minimierung einer Wirkung. Beide Deutungen sind völlig äquivalent, beide sind in der Physik akzeptiert, und je nach Problemstellung benutzt man mal die eine, mal die andere.

    Nochmals: Du hast offensichtlich keine Ahnung, wovon du redest.

  132. #132 MartinB
    8. September 2015

    @Bjoern
    Gutes Beispiel mit dem Prinzip der kleinsten Wirkung.
    Man könnte gleich noch den Hamilton-Formalismus dazunehmen – auch in der klassischen Physik gibt es damit immerhin drei äquivalente Formulierungen.
    Auch heisenberg- vs. Schrödinger-Bild passen eigentlich zumindest halbwegs in diese Kategorie.

  133. #133 MartinB
    8. September 2015

    @Bjoern
    Noch ein viel schöneres beispiel ist mir gerade eingefallen (hätte ich wirklich eher dran denken können, hab ich schließlich viel drüber gebloggt) Die Deutung von G_munu in der ART als Raumzeitmetrik oder Feld. Zwei vollkommen andere sichtweisen, mathematisch äquivalent, aber auf ganz anderen Konzepten basierend, und beide werden benutzt. (Wobei sich zwischen beiden entscheiden ließe, wenn jemand ein Wurmloch zwischen zwei gegebenen raumpunkten öffnen könnte, ichglaube, das geht im Feld-Modell nicht).

  134. #134 a.n
    9. September 2015

    Ob ich Bewegungsgleichungen nehme oder die entsprechende Wirkung — das ist äquivalent und unterschiedliche Deutungen sehe ich da nicht, was soll denn damit gemeint sein? Anders ist es bei Deutungen der Quantenmechanik, das ist richtig, dahinter stecken ja unterschiedliche Ontologien.

    “Die Deutung von G_munu in der ART als Raumzeitmetrik oder Feld.” Was ist damit gemeint? Der Einsteintensor (keine Metrik, sondern das “Spurinverse” der Ricci-Krümmung und damit verantwortlich für die Expansion von geodätischen Bällen) ist doch ein Feld? Ein Schnitt im Vektorbündel.

  135. #135 MartinB
    9. September 2015

    @a.n.
    Gemeitn ist, dass es konzeptionell einen großen Unterschied macht, ob ich von einem prinzip der kleinsten Wirkung oder einer Bewegungsgleichung ausgehe – man sieht’s letztlich auch am Übergang zur QM, wo man entsprechend mit dem Prinzip der kleinsten Wirkung beim Pfadintegral landet (und das Prinzip so auch anschaulich verstehen kann).

    Feld oder Metrik ist so gemeint, dass die ART sowohl als Theorie der Raumzeitkrümmung (also mit dynamischer Raumzeit) interpretiert werden kann als auch als Theorie auf einer flachen Minkowski-Metrik, auf der sich eben ein entsprechendes feld befindet (so wie es die Stringtheoretikerinnen machen). Beides ist in jeder Hinsicht äuivalent, siehe auch meine entsprechenden texte oder z.B. MisnerThorneWheeler Kap. 18.

  136. #136 Bjoern
    9. September 2015

    @a.n: Auch wenn es mathematisch äquivalent ist – konzeptuell ist es ein sehr großer Unterschied, ob man sagt “das Teilchen folgt dieser Bahn, weil an dem und dem Punkt die und die Kräfte auf es einwirken” oder ob man sagt “das Teilchen folgt dieser Bahn, weil auf dieser die Wirkung minimal ist”. Das fängt schon damit an, dass das erste eine lokale, das zweite aber eine globale Sichtweise der Teilchenbewegung ist.

    @Martin: Hattest du nicht vor ein paar Jahren sogar mal genau dazu einen Artikel geschrieben? Ich dachte, ich hätte das von dir – dein Lob in #132 ist also unnötig.

  137. #137 MartinB
    10. September 2015

    @Bjoern
    Ja, hatte ich – aber ich hatte halt nicht mehr dran gedacht, das Lob ist also voll verdient…

  138. #138 Bjoern
    10. September 2015

    @Martin: Apropos alternative, mathematisch äquivalente Darstellungen der ART: Kennst du diesen Artikel schon?
    https://arxiv.org/pdf/gr-qc/0103044v6.pdf

  139. #139 Benjamin Packisch
    10. September 2015

    @MartinB.

    “….er blendet die Möglichkeit (z.B. evaneszenter) E-Feldstreuung in den B-Faden aus. ”
    Ja, und wenn er trotzdem zum richtigen Ergebnis kommt, dann ist
    1. dieser Weg damit doch als mögliche Erklärung etabliert …

    Bitte was? Weil er es nicht erwähnt, ist es als mögliche Erklärung etabliert!? Und weil dann damit das erwartete Ergebnis herausbekommt, soll es “richtig” sein ?! Das nun wirklich ein hanebüchener Zirkelschluss…aber er erhält zugegebenermaßen die Binnenlogik hier – und er entbindet von ausdrücklichen Willensäußerungen.

    Zu Kant und der Freiheit ist vielleicht seine dritte Antinomie der reinen Vernunft interessant. Über Kant hinaus geht dann die Frage, wie wir mit diesen Widersprüchen umgehen. Deine Antwort und Deinen Anspruch, damit die Physik als Fachgebiet zu repräsentieren, habe ich nun zur Kenntnis genommen.
    Allein, ich will weiterhin dazu ermuntern, den Konsens der Schulphysik mutig und logisch sauber zu transzendieren. Ihn zu verteidigen ist zwar einfacher, aber das bringt uns nicht weiter.

    In diesem Sinne
    BP

  140. #140 Bjoern
    10. September 2015

    “Schulphysik”? Aua.

  141. #141 MartinB
    10. September 2015

    @Benjamin
    “Dieser Weg” bezog sich auf die Erklärung über das Vektorpotential.

    Und was die 3. Antinomie angeht – wie gesgat, die ist mir nicht unbekannt.

    “Deine Antwort und Deinen Anspruch, damit die Physik als Fachgebiet zu repräsentieren”
    Ich repräsentiere gar nichts – außer meinem Verständnis der Physik.

    ” den Konsens der Schulphysik mutig und logisch sauber zu transzendieren. ”
    Dann mach mal. Von logisch sauber war bisher ja nicht viel zu sehen. Wir warten immer noch, und solange dunicht mal was Substantielles beiträgst (wie die “evaneszente” Wewi denn nun funktionieren sol z.B.) hat das hier wenig Sinn. “Ich glaube ganz fest, dass es auch eine alternative Physik geben kann, kann aber keine Beispiele nennen” und “Ich glaube, dass die böse schulphysik Alternativen nicht zulässt und akzeptriere keine gegenbeispiele (und gehe vorsichtshalber nicht drauf ein, wenn welche gebracht werden)” ist beides keine Grundlage für eine sinnvolle Diskussion (und mit “Logik” hat das schon gar nichts zu tun…)

    “Ihn zu verteidigen”
    Wogegen? Bisher hast du ja nirgends eine sinnvolle Alternative aufzeigen können.

  142. #142 Benjamin Packisch
    11. September 2015

    @MartinB
    “Dieser Weg” bezog sich auf die Erklärung über das Vektorpotential. ”
    Das ist wirklich grotesk, wie wenig das zu dem AB-Artikel passt, oder zu meinem Einwand… auch ein bisschen lustig aus der Entfernung.

    Ich habe genügend Alternativaspekte hier aufgezeigt, und Du hast daraufhin immer gezeigt, dass Du diese Alternativen nicht akzeptieren wirst, egal wie oder womit ich argumentiere. Ergo: Mit Argumenten bist Du nicht erreichbar für neue Sichtweisen, weil Dich die Üblichkeiten offenbar so fesseln.
    Nun gut – dafür, dass Du nach Deinen Worten gar nichts “verteidigen” musst oder willst, zeigst Du erstaunliche Beharrungsbereitschaft.

    Wohlgemerkt habe ich nie angezweifelt, dass die konventionelle physikalische Herangehensweise möglich wäre – ich habe stattdessen in der hier geboteten Form skizziert, dass sie nicht alleinseligmachend ist und für den wendigen und mutigen Selbstdenker Alternativen möglich sind. Wer berufen ist, wird hier jetzt genügend Hinweise finden.

    Dir wünsche ich auch weiterhin viel Vergnügen in den erprobten Gefilden, auch mit den DInos (nur den harmlosen, selbstredend, “wer wollte denn echte Drachen erlegen…?”)!
    BP

  143. #143 Bjoern
    11. September 2015

    @Benjamin: Du hast hier keinerlei “Alternativaspekte aufgezeigt”. Du hast nur drauflos spekuliert in der Art “Aber es könnte doch auch ganz anders sein!!!!!einself”, ohne im Mindesten zu zeigen, dass diese andere Möglichkeit auch tatsächlich funktioniert (sprich: mit den Beobachtungen übereinstimmt). Und zusätzlich hast du gezeigt, dass du noch nicht einmal die konventionelle Erklärung verstanden hast.

    Kurz gesagt: Du hast dich hier nach Strich und Faden lächerlich gemacht.

  144. #144 MartinB
    11. September 2015

    @Benjamin
    Wie Bjoern schon sagte: “Ich kenne einen E-techniker, der sich vorstellen kann, dass…” ist keine physikalische Alternative, ebensowenig wie “Es könnte doch auch eine evaneszente Welle sein”, ohne im mindesten zu erklären, wo die wie und warum abklingen soll und wie sich mit so einer Welle ein wegunabhängiges Resultat erzielen lässt.
    Nochmal das Argument von oben, da es dir ja anscheinend schwer fällt, es zu verstehen:
    1. Du argumentierst mit einer möglichen WeWi des elektrischen feldes des Elektrons mit dem Flussschlauch.
    2. Ich zeige, dass die herkömmliche Berechnung des AB-Effekts die Eigenschaft hat
    a. mit dem Experiment in verschiedenen Fällen überinzustimmen und
    b. wegunabhängig zu sein, also dasselbe Resultat zu liefern, egal wie man das elektron um die Spule führt.
    Daraus schließe ich, dass der AB-Effekt tatsächlich wegunabhängig ist und dass jede alternative Erklärung diese Eigenschaft haben muss (was eine Erklärung über E-Felder nicht tut).
    Folgerung: Wenn du nach wie vor beharrst, Deine Idee sei eine Alternative, musst du zeigen, wie sie zum richtigen Ergebnis kommt, obwohl sie nicht wegunabhängig ist oder wie du sie wegunabhängig formulieren kannst. Beides hast du nicht getan.
    Stattdessen präsentierst du hier herablassende Bemerkungen der Art “Ihr denkt ja nur in eingefahrenen Schienen”, und sagst Dinge, die vollkommen jeder Korrektheit entbehren (wie etwa, dass die SGL nur für das H-Atom ohne Zusatzannahmen funktioniert oder dass da noch Extra-Annahmen drinstecken, weil deiner Ansicht nach Teilchen im Grundzustand Energie durch Strahlung verlieren können.)

    Wie sagt man so schön
    “keep an open mind, but not so open your brain falls out”

  145. #145 AmbiValent
    29. November 2015

    @MartinB

    Hier noch einmal im “richtigen” Artikel mein letzter kürzerer Kommentar mit Frage an dich und andere, wie ihr darüber denkt:

    Die IAU könnte beschließen, dass die erste Interpretation gilt und dass ein Objekt, bei dem das Equilibrium erwiesen ist, zum Zwergplaneten erklärt werden kann. Oder aber, dass die zweite Interpretation gilt und dass ein Objekt, bei dem man eine bestimmte Mindestmasse nachgewiesen hat, zum Zwergplaneten erklärt werden kann. Oder aber als dritte Option gar nichts beschließen.

    Ich bin für die zweite Option. Und wenn sich die IAU für die dritte Option entscheidet, wüsste ich gerne den Grund dafür. (Fehlt uns das notwendige Wissen noch, das wir bräuchten, um eine Entscheidung zu treffen? Wenn ja, in welcher Zeitspanne ist zu erwarten, dass wie dies herausfinden?)

    Frage an Martin und andere Leser: wie seht ihr das?

  146. #146 Krypto
    29. November 2015

    @AmbiValent:
    Hier ist Dein Thema aber im “falschen” Artikel 😉